Theory of Equation Book Du

Download as pdf or txt
Download as pdf or txt
You are on page 1of 110

Appendix-112

Resolution No. 38 {38-1 [38-1-13(6)]}

Generic Elective (GE-1)


(for Hons. other than Mathematics)
Semester-I
Course Credit - 4

As per the UGCF - 2022 and National Education Policy 2020


Editorial Board
Prof. S.K. Verma
Ms. Mridu Sharma

Content Writers
Ms. Setu Rani
Dr. Neha Bhatia

Academic Coordinator
Deekshant Awasthi

© Department of Distance and Continuing Education

Ist edition: 2022


E-mail: [email protected]
[email protected]

Published by:
Department of Distance and Continuing Education under
the aegis of Campus of Open Learning, University of Delhi
Printed by:
5, Cavalry Lane, School of Open Learning, University of Delhi,
Delhi-110007
Contents

1 General Properties of Polynomial Equations 3


1.1 Learning Objectives . . . . . . . . . . . . . . . . . . . . . . . . . . . . . . 4
1.2 Introduction . . . . . . . . . . . . . . . . . . . . . . . . . . . . . . . . . . 4
1.3 Polynomials . . . . . . . . . . . . . . . . . . . . . . . . . . . . . . . . . . 4
1.4 General Properties of Polynomials and Equations . . . . . . . . . . . . . . 5
1.5 Fundamental Theorem of Algebra and its Consequences . . . . . . . . . . 15
1.6 Theorems on imaginary, integral and rational roots . . . . . . . . . . . . . 15
1.7 Descartes’ rule of signs . . . . . . . . . . . . . . . . . . . . . . . . . . . . 21
1.8 Summary . . . . . . . . . . . . . . . . . . . . . . . . . . . . . . . . . . . 25
1.9 Self-Assessment Exercises . . . . . . . . . . . . . . . . . . . . . . . . . . 26
1.10 Solutions to In-text Exercises . . . . . . . . . . . . . . . . . . . . . . . . . 26

2 Relation Between the Roots and Coefficients of Equations 30


2.1 Learning Objectives . . . . . . . . . . . . . . . . . . . . . . . . . . . . . . 30
2.2 Introduction . . . . . . . . . . . . . . . . . . . . . . . . . . . . . . . . . . 30
2.3 Relation Between Roots and Coefficients of Equations . . . . . . . . . . . 31
2.4 Applications to solution of Equations . . . . . . . . . . . . . . . . . . . . 36
2.5 Summary . . . . . . . . . . . . . . . . . . . . . . . . . . . . . . . . . . . 44
2.6 Self Assessment Exercise . . . . . . . . . . . . . . . . . . . . . . . . . . . 45
2.7 Solutions to In-text Exercises . . . . . . . . . . . . . . . . . . . . . . . . . 46

3 De Moivre’s Theorem and its Applications 48


3.1 Learning Objectives . . . . . . . . . . . . . . . . . . . . . . . . . . . . . . 48
3.2 Introduction . . . . . . . . . . . . . . . . . . . . . . . . . . . . . . . . . . 49
3.3 Complex Numbers . . . . . . . . . . . . . . . . . . . . . . . . . . . . . . 49
3.4 De Moivre’s Theorem . . . . . . . . . . . . . . . . . . . . . . . . . . . . . 54
3.5 Applications of De Moivre’s Theorem . . . . . . . . . . . . . . . . . . . . 60
3.6 nth Roots of Unity . . . . . . . . . . . . . . . . . . . . . . . . . . . . . . 66
3.7 Summary . . . . . . . . . . . . . . . . . . . . . . . . . . . . . . . . . . . 68
3.8 Self-Assessment Exercises . . . . . . . . . . . . . . . . . . . . . . . . . . 69
3.9 Solutions to In-text Exercises . . . . . . . . . . . . . . . . . . . . . . . . . 69

4 Cubic and Biquadratic Equations 73


4.1 Learning Objectives . . . . . . . . . . . . . . . . . . . . . . . . . . . . . . 73
4.2 Introduction . . . . . . . . . . . . . . . . . . . . . . . . . . . . . . . . . . 74

1
2 CONTENTS

4.3 Algebraic solution of the cubic equation . . . . . . . . . . . . . . . . . . . 74


4.4 Transformation of Equations . . . . . . . . . . . . . . . . . . . . . . . . . 79
4.5 Summary . . . . . . . . . . . . . . . . . . . . . . . . . . . . . . . . . . . 88
4.6 Self Assessment Exercise . . . . . . . . . . . . . . . . . . . . . . . . . . . 88
4.7 Solutions to In-text Exercises . . . . . . . . . . . . . . . . . . . . . . . . . 89

5 Symmetric Functions 90
5.1 Learning Objectives . . . . . . . . . . . . . . . . . . . . . . . . . . . . . . 90
5.2 Introduction . . . . . . . . . . . . . . . . . . . . . . . . . . . . . . . . . . 91
5.3 Symmetric Functions . . . . . . . . . . . . . . . . . . . . . . . . . . . . . 91
5.4 Fundamental Theorem on Symmetric Functions . . . . . . . . . . . . . . . 94
5.5 Rational Functions Symmetric in all but One of the Roots. . . . . . . . . . 95
5.6 Sums of Like Powers of the Roots . . . . . . . . . . . . . . . . . . . . . . 96
5.7 Newton’s Theorem on the Sums of the powers of the roots . . . . . . . . . 97
5.8 Theorems relating to Symmetric Functions . . . . . . . . . . . . . . . . . . 98
5.9 Computation of Symmetric Functions . . . . . . . . . . . . . . . . . . . . 100
5.10 Summary . . . . . . . . . . . . . . . . . . . . . . . . . . . . . . . . . . . 100
5.11 Self Assessment Excercise . . . . . . . . . . . . . . . . . . . . . . . . . . 100

6 Transformation by Symmetric Functions 101


6.1 Learning Objectives . . . . . . . . . . . . . . . . . . . . . . . . . . . . . . 101
6.2 Introduction . . . . . . . . . . . . . . . . . . . . . . . . . . . . . . . . . . 101
6.3 Transformation by Symmetric Functions . . . . . . . . . . . . . . . . . . . 102
6.4 Transformation in General . . . . . . . . . . . . . . . . . . . . . . . . . . 104
6.5 Equation of Differences in General . . . . . . . . . . . . . . . . . . . . . . 106
6.6 Summary . . . . . . . . . . . . . . . . . . . . . . . . . . . . . . . . . . . 108
6.7 Self Assessment Exercise . . . . . . . . . . . . . . . . . . . . . . . . . . . 108
Lesson - 1

General Properties of Polynomial


Equations

Structure
1.1 Learning Objectives 4
1.2 Introduction 4
1.3 Polynomials 4
1.3.1 Polynomials 4
1.3.2 Degree of a Polynomial 5
1.4 General Properties of Polynomials and Equations 5
1.4.1 The Remainder Theorem 5
1.4.2 The Factor Theorem 8
1.4.3 Synthetic Division 10
1.4.4 Factored form of a polynomial 13
1.4.5 Multiple Roots 14
1.5 Fundamental Theorem of Algebra and its Consequences 15
1.6 Theorems on imaginary, integral and rational roots 15
1.6.1 Newton’s method for integral roots 19
1.6.2 Rational Roots 20
1.7 Descartes’ rule of signs 21
1.7.1 Descartes’ rule of signs-Positive roots 21
1.7.2 Descartes’ rule of signs-Negative roots 22
1.7.3 Application of Descartes’ rule of signs for finding the Imaginary
roots 24
1.8 Summary 25
1.9 Self-Assessment Exercises 26
1.10 Solutions to In-text Exercises 26

3
4 LESSON - 1. GENERAL PROPERTIES OF POLYNOMIAL EQUATIONS

1.1 Learning Objectives


Student will be able to

• divide a polynomial by a linear polynomial without performing actual division.

• find the number of positive, negative and imaginary roots of a polynomial without
actually solving it.

• understand the basic properties of roots of a polynomial equation.

1.2 Introduction
One of the oldest problems in mathematics is solving algebraic equations, in particular find-
ing the roots of the polynomial equations. The ancient mathematicians solved some partic-
ular problems and there was no generality. In this chapter we shall study some important
theorems related to polynomial equations such as the Remainder theorem, Factor theorem,
Fundamental theorem of algebra, etc. “The Fundamental theorem of algebra” which state
that every polynomial of degree ≥ 1 has at least one zero was first proved by the famous
German mathematician Karl Fredrich Gauss. Also we will learn about Descartes’ rule of
signs for counting the positive and negative roots of a polynomial equation without actually
finding the roots. Using these ideas we will reach our goal of solving polynomial equations
of certain type. Here we will also understand some properties of the roots of a polynomial
equation. But before discussing about these theorems, we start a brief discussion of the
polynomial equations and degree of the polynomial.

1.3 Polynomials
1.3.1 Polynomials
Polynomials is an algebraic expression that consist of variables (also called indeterminate)
and coefficient. It involves only the operation of addition, subtraction, multiplication and
non negative integer exponents(power) of variables.

Standard form of a polynomial is a0 xn + a1 xn−1 + a2 xn−2 .......an , where a0 , a1 , .....an are


real constants called coefficients of the polynomial and x is a variable.

Example 1.1. 1. 4x−3 + 5 is not a polynomial because x−3 have negative exponent (i.e.
−3).

2. 5x2 + 2y − 7 is a polynomial in two variables( x and y) and 5, 2 are the coefficient of


x2 and y respectively while −7 is constant.

3. 8x2 − 5x + 6 is a polynomial in one variables x and 8, −5 are the coefficient of x2


and x respectively while 6 is constant.
1.4. GENERAL PROPERTIES OF POLYNOMIALS AND EQUATIONS 5

Figure 1.1: Polynomial with three terms.

1.3.2 Degree of a Polynomial


Degree of a polynomial is simply the highest exponent occurs in the polynomial equation.
The degree of a term is the sum of the exponents of the variables that appears in it.
Example 1.2. 1. The polynomial 7x3 y 3 +4x2 −9x+y has four terms. The first term has
degree 6(3+3), the second term has degree 2 , third term has degree 1 and fourth term
has degree 1. Therefore the polynomial has degree 6, which is the highest degree of
any term.
2. x3 − 5x2 + 6x + 2 has degree 3.
3. x2 y + 5x2 + 3 has degree 3
Definition 1.1. An equation of the form
p(x) ≡ a0 xn + a1 xn−1 + a2 xn−2 .......an = 0 (1.1)
is called a polynomial equation of degree n.
Remark. 1. The polynomial equation of degree one, a0 x + a1 = 0 is called linear
equation.
2. The polynomial equation of degree two, a0 x2 + a1 x + a2 = 0 is called quadratic
equation.
3. The polynomial equation of degree three, a0 x3 + a1 x2 + a2 x + a3 = 0 is called cubic
equation.
4. The polynomial equation of degree four, a0 x4 + a1 x3 + a2 x2 + a3 x + a4 = 0 is called
bi quadratic equation.

1.4 General Properties of Polynomials and Equations


1.4.1 The Remainder Theorem
The remainder theorem or the polynomial remainder theorem is used to find the remainder
when a polynomial is divided by a linear polynomial without actually carrying out the steps
6 LESSON - 1. GENERAL PROPERTIES OF POLYNOMIAL EQUATIONS

of the long divisions. It states that

Theorem 1.1 (The Remainder Theorem). When a polynomial p(x) (whose degree is greater
than or equal to one) is divided by a linear polynomial (x−c) until a remainder independent
of x is obtained then the remainder is given by p(c) ( which is a value of p(x) when x = c.)

Proof. Here dividend is p(x) and divisor is x − c. Let quotient is q(x) and remainder is
denoted by r. As we know

divident = divisor × quotient + remainder. (1.2)

Then by using the dividend formula (1.2), we have

p(x) = (x − c) q(x) + r (1.3)

By taking x = c in equation (1.3), we get p(c) = r.

Remark. 1. This theorem is also known as little Bézout’s theorem

2. This theorem works only when the divisor is linear ( this can be considered as one of
its limitation)

Example 1.3. Without actual division , find the remainder when x4 − 3x2 − x − 6 is divided
by x + 3.

Solution. Given a polynomial

p(x) = x4 − 3x2 − x − 6. (1.4)

To find the remainder here, we need not to carry out the long steps of division. Here the
divisor is x + 3 and dividend is x4 − 3x2 − x − 6.
Let

x + 3 = 0 then x = −3 (1.5)

By putting x = −3 from equation (1.5) in the given polynomial (1.4), we get

p(−3) =(−3)4 − 3(−3)2 − (−3) − 6


=81 − 27 + 3 − 6
=51.
1.4. GENERAL PROPERTIES OF POLYNOMIALS AND EQUATIONS 7

Verification:
x3 −3x2 +6x −19
x + 3 ) x4 −3x2 −x −6
x4 +3x3
−3x3 −3x2 −x −6
−3x3 −9x2
6x2 −x −6
6x2 +18x
−19x −6
−19x −57
51

Thus by dividing the given polynomial (1.5) with x + 3, we get the value of remainder as
51.
Example 1.4. Find the remainder when p(x) = 3x3 + x2 + 2x + 5 is divided by x + 1.
Solution. Given a polynomial

p(x) = 3x3 + x2 + 2x + 5 (1.6)

and divisor is x + 1. Then by putting

x + 1 = 0 we get x = −1 (1.7)

So, the remainder is calculated by substituting x = −1 from (1.7) into equation (1.6).

p(−1) =3(−1)3 + (−1)2 + (−1) + 5


=−3+1−2+5
=1

Thus, by dividing the given polynomial (1.6) with x + 1, we get the value of remainder as 1.

Verification:
3x2 −2x +4
x + 1 ) 3x3 +x2 +2x +5
3x3 +3x2
−2x2 +2x +5
−2x2 −2x
4x +5
4x +4
1
8 LESSON - 1. GENERAL PROPERTIES OF POLYNOMIAL EQUATIONS

1.4.2 The Factor Theorem


The Factor theorem is a theorem which gives the relation between the factors and zeros of
the polynomial. It states that

Theorem 1.2 (The Factor Theorem). A polynomial p(x) has a factor (x − c) if and only if
p(c) = 0 ( i.e. c is the root of the polynomial p(x) .

Or

If c is the root of the polynomial equation p(x) = 0 i.e. p(c) = 0 then (x − c) is the factor
of p(x).

Proof. Consider a polynomial p(x) which has x−c as one of its factor. Then from equation
(1.3) in previous theorem 1.1 we have

p(x) = (x − c) q(x) + p(c) (1.8)

Since x − c is one of the factor of p(x), therefore using equation (1.8) one can conclude
that the value of remainder must be zero i.e. p(c) = 0.

Converse: Since it is given that p(c) = 0, therefore from equation (1.8) we have

p(x) = (x − c) q(x)

Thus (x − c) is one of the factor of p(x).

Remark. Following statements are equivalent for any polynomial p(x)

1. The remainder is zero when p(x) is exactly divided by (x − c).

2. c is the solution of p(x).

3. (x − c) is a factor of p(x).

4. c is the root of the polynomial p(x) i.e. p(c) = 0.

Remark. Factor theorem is a special case of the polynomial remainder theorem. i.e. when
r = 0 in equation (1.3), then p(x) = (x − c) q(x)

Example 1.5. Without actual division show that 2x4 − x3 − 6x2 + 4x − 8 is divisible by
x + 2 or x + 2 is one of the factor of 2x4 − x3 − 6x2 + 4x − 8.

Solution. Consider the polynomial

p(x) = 2x4 − x3 − 6x2 + 4x − 8. (1.9)


1.4. GENERAL PROPERTIES OF POLYNOMIALS AND EQUATIONS 9

To show that p(x) is divisible by x + 2 or x + 2 is one of the factor of p(x), we have to show
p(−2) = 0 (since x + 2 = 0 then x = −2). By putting x = −2 in the given polynomial
(1.9), we get

p(−2) =2(−2)4 − 2(−2)3 − 6(−2)2 + 4(−2) − 8


=32 + 8 − 24 − 8 − 8
=0

Since the value of remainder is zero, therefore x + 2 is one of the factor of 2x4 − x3 − 6x2 +
4x − 8.

Example 1.6. Check whether 2x − 1 is one of the factor of 2x3 − x2 − 2x + 1 or x = 12 is


the zero of the polynomial equation 2x3 − x2 − 2x + 1 = 0. Also verify the statement.

Solution. Since 2x − 1 is the divisor of the polynomial

p(x) = 2x3 − x2 − 2x + 1, (1.10)

then by putting
1
2x − 1 = 0 we get x = (1.11)
2

   3  2  
1 1 1 1
p =2 − −2 +1
2 2 2 2
1 1
= − −1+1
4 4
=0
1
Hence (2x − 1) is one of the factor of 2x3 − x2 − 2x + 1 or x = 2
is the zero/solution/root
of the polynomial 2x3 − x2 − 2x + 1.

Verification:

x2 −1
2x − 1 ) 2x3 −x2 −2x +1
2x3 −x2
−2x +1
−2x +1
0

Hence (2x − 1) is one of the factor of 2x3 − x2 − 2x + 1.

In-text Exercise 1.1. Solve the following questions:


10 LESSON - 1. GENERAL PROPERTIES OF POLYNOMIAL EQUATIONS

(a) Without actual division find the remainder when x3 − 3x2 + 6x − 5 is divided by
x − 3.

(b) Without actual division find the remainder when 4x3 − 17x2 + 9 is divided by 3x − 5.

(c) Find the value of k if 4x3 − 2x2 + kx + 5 leaves remainder −10 when divided by
2x + 1.

(d) Without actual division show that

(i) 18x10 + 19x5 + 1 is divisible by x + 1.


(ii) r3 − 1, r4 − 1, r5 − 1 are divisible by r − 1.

(e) Check whether 7 + 3x is a factor of 3x3 + 7x.

1.4.3 Synthetic Division


Synthetic Division is a shortcut way for the long Division. As we know long division with
polynomials involve many steps, synthetic divisions carries the calculations even in a few
steps
Here we learn synthetic divisions by linear polynomial. The advantage of this method over
the remainder theorem and factor theorem is that using synthetic division we can find the
quotient and remainder both of the given polynomial. Here we will understand the steps of
synthetic division by taking an example.

Example 1.7. Consider a polynomial p(x) = x4 + 3x3 − 2x − 5 is divided by x − 2. What


is the quotient and remainder in this case.

Step-1: Write the coefficients of the polynomial in descending order in a line by ignoring
the powers of x. Also write the zero coefficient of missing powers of x.

Implication Since the coefficient of x4 is 1, x3 is 3, x2 is 0, x is −2 and the value of


constant is −5.

∗ 1 3 0 −2 −5

Step-2: Write the value of linear polynomial by which we have to divide the given polyno-
mial. Here we have to divide p(x) by x − 2 = 0 i.e. x = 2. The value of x is written at the
place of ∗ in table of step-1.

Implication
1.4. GENERAL PROPERTIES OF POLYNOMIALS AND EQUATIONS 11

2 1 3 0 −2 −5

Step-3: Write the same coefficient a0 below in the third line

a a0 a1 a2 a3 a4

a0

Implication

2 1 3 0 −2 −5

Step-4: Multiple a0 by a and then write it below the entry a1 in second row. Then add a1
and a0 a and write it in the third row. Again multiply a1 + a0 a by a and then write it below
the entry a2 . Then add a2 and a(a0 a + a1 ) and write it in the third row and follow the same
procedure for all the entries.

a a0 a1 a2 a3 a4
a0 a a(a0 a + a1 )
a0 a0 a + a1 a(a0 a + a1 ) + ...
a2

Implication Multiply 2 by 1 and write it below 3.

2 1 3 0 −2 −5
2
1

add both the values and write it in the third row.

2 1 3 0 −2 −5
2
1 5
12 LESSON - 1. GENERAL PROPERTIES OF POLYNOMIAL EQUATIONS

Again multiple 5 by 2 then write it below 0. Add both the number and write it in the third
row.

2 1 3 0 −2 −5
2 10
1 5 10

By following the same procedure we get the following table

2 1 3 0 −2 −5
2 10 20 36
1 5 10 18 31

Remark. 1. Last entry in the third row represent the remainder of the polynomial after
dividing it by a linear factor.

2. Quotient is given by multiplying the entry of third row with one less degree equation.
For example, in the above question 31 is the remainder while quotient is 1.x3 +5.x2 +
10.x + 18.

3. We have to carry out all the steps of this synthetic division in a single table.

Example 1.8. Divide x3 + 3x2 − 2x − 5 by x − 2.

Solution. Here x − 2 = 0 implies x = 2. Hence

2 1 3 −2 −5
2 10 16
1 5 8 11

the remainder is 11 and the value of quotient is 1.x2 + 5.x + 8 = x2 + 5x + 8.

Example 1.9. Divide x3 + x2 + x + 1 by x + 1

Solution. Here x + 1 = 0 implies x = −1. Hence

−1 1 1 1 1
−1 0 −1
1 0 1 0

the remainder is 0 and the value of quotient is 1.x2 + 0.x + 1 = x2 + 1.


1.4. GENERAL PROPERTIES OF POLYNOMIALS AND EQUATIONS 13

In-text Exercise 1.2. Solve the following questions:

(a) Divide x3 + 6x2 + 10x − 1 by x − 3.

(b) Without actual division find the remainder when 3x3 − 17x2 − x + 15 is divided by
3x − 5.

(c) Find the quotient of 2x4 − x3 − 6x2 + 4x − 8 by x2 − 4.

1.4.4 Factored form of a polynomial


Polynomials can be written in the factored form. The factored form of a polynomial means
it is written as a product of its factors.
Consider a polynomial of degree ‘n’ whose leading coefficients (coefficient of highest de-
gree term) is not zero.

p(x) ≡ a0 xn + a1 xn−1 + a2 xn−2 + ... + an (an ̸= 0). (1.12)

If p(x) = 0, has the root α1 (either real or imaginary), then by using factor theorem 1.2,
p(x) has one of the factor (x − α1 ). So the polynomial equation (1.12) can be written as

p(x) ≡ (x − α1 )g(x), where g(x) ≡ a0 xn−1 + a′1 xn−2 + ... + a′n−1

If g(x) = 0 has the root α2 , then

g(x) ≡ (x − α2 )g1 (x), where g1 (x) ≡ a0 xn−2 + b′1 xn−3 + ... + b′n−2 .

Therefore

p(x) ≡ (x − α1 )(x − α2 )g1 (x)

If g1 (x) = 0 has the root α3 , then continuing in the same way as above we get

p(x) ≡ a0 (x − α1 )(x − α2 )(x − α3 )g2 (x)

Proceeding in the same way we get

p(x) ≡ a0 (x − α1 )(x − α2 )(x − α3 )...(x − αn ). (1.13)

From the above discussion, we can conclude that if any equation p(x) = 0 of degree ‘n’
is known to have ‘n’ distinct roots α1 , α2 ...αn , then p(x) can be expressed in the factored
form (1.13).

Theorem 1.3. An equation of degree ‘n’ cannot have more than ‘n’ distinct roots.
Proof. Since a polynomial equation (1.12), p(x) = 0 of degree ‘n’ having ‘n’ roots α1 ,
α2 ...αn can be written in the factored form as (1.13)

p(x) ≡ a0 (x − α1 )(x − α2 )(x − α3 )...(x − αn ). (1.14)


14 LESSON - 1. GENERAL PROPERTIES OF POLYNOMIAL EQUATIONS

Let equation (1.12) has one more root α different from α1 , α2 ...αn of polynomial equation
p(x). since α is the root of p(x) = 0, therefore p(α) = 0. Hence from equation (1.14)

0 = p(α) ≡ a0 (α − α1 )(α − α2 )(α − α3 )...(α − αn ). (1.15)

Thus, one of the factor of right hand side of equation (1.15) must be zero say

α − αi = 0
α = αi

Hence any polynomial of degree ‘n’ cannot have more than ‘n’ distinct roots.

1.4.5 Multiple Roots


Multiple roots of a polynomial are roots whose factors show up more than once in the
complete factorization of the polynomial. The number of times a factor shows up in the
complete factorization is called the multiplicity of the corresponding root.
Consider in equation (1.12), α1 has multiplicity m1 i.e. α1 is the root repeating m1 times.
Also multiplicity of α2 is m2 and so on. Then equation (1.12) becomes

p(x) ≡ a0 (x − α1 )m1 (x − α2 )m2 ...(x − αk )mk (1.16)

where m1 + m2 + ... + mk = n = degree of the polynomial. Also α1 , α2 ...αk are distinct


roots of the polynomial.

Theorem 1.4. An equation of degree n cannot have more than n roots, a root of multiplicity
m being counted as m roots.
Proof. Let α1 has multiplicity m1 , α2 has multiplicity m2 ,.. and αk has multiplicity mk .
Then

p(x) ≡ a0 (x − α1 )m1 (x − α2 )m2 ...(x − αk )mk (1.17)

where m1 + m2 + ... + mk = n = degree of the polynomial. Also α1 , α2 ...αk are distinct


roots of the polynomial. Since (x − α1 )m1 is a factor of p(x) or p(x) is exactly divisible by
(x − α1 )m1 , let us assume equation (1.17) has one more root α1 . Then multiplicity of α1
becomes m1 + 1. Thus

p(x) ≡ a0 (x − α1 )m1 +1 (x − α2 )m2 ...(x − αk )mk (1.18)

Since, p(x) is exactly divisible by (x − α1 )m1 , therefore it will not be exactly divisible by
(x − α1 )m1 +1 . (because α1 is a root of multiplicity m1 .) An equation of degree n cannot
have more than n roots, a root of multiplicity m being counted as m roots.

Example 1.10. Let the polynomial equation p(x) = 5(x − 2)(x − 6)2 (x − 3)3 (x − 4)5 .
Then 2 is the simple root, 6 is the root with multiplicity 2, 3 is the root with multiplicity
3 and 4 is the root with multiplicity 5 of given equation of degree 11 which has no further
roots.
1.5. FUNDAMENTAL THEOREM OF ALGEBRA AND ITS CONSEQUENCES 15

1.5 Fundamental Theorem of Algebra and its Consequences


Fundamental theorem of algebra also known as d’Alembert’s theorem states that

Theorem 1.5. Every non constant single variable polynomial or every algebric equation
with complex coefficients has at least one complex (real and imaginary) roots

or

Any polynomial of degree n has n roots.

Let us consider p(x) = a0 xn + a1 xn−1 ... + an = 0 be any equation of degree n where


a0 ̸= 0. Since by using fundamental theorem of algebra, every equation of degree n ≥ 1
has at least one root. Therefore using factored form of polynomial, it will have exactly n
roots of the polynomial.

Remark. This theorem does not reveal what the roots of the polynomial equation are but
it tells about the number of roots of an equation.

Example 1.11. How many total roots exist for the function p(x) = −6x2 + x3 − 6 + 11x.

Solution. Since the highest degree of the given polynomial is 3, therefore it will have
exactly 3 roots. On solving p(x) = −6x2 + x3 − 6 + 11x = 0, we get the roots as 1, 2, 3.

1.6 Theorems on imaginary, integral and rational roots


Theorem 1.6. The complex roots occurs in conjugate pair.

Or

In a given equation

a0 xn + a1 xn−1 + ... + an = 0 and a0 ̸= 0 (1.19)

where a0 , a1 , ...an are coefficients of the equation and the value of all the coefficients are
real constant number. If the given polynomial has a complex root a + ib then it will also
has the root a − ib (i.e. its complex conjugate will also be the root of the equation.)

Proof. Proof of the theorem is omitted.

Example 1.12. Find the roots of the polynomial equation x3 + x2 + x + 1 = 0.

Solution. The given equation is

p(x) = x3 + x2 + x + 1 = 0. (1.20)

Since −1 satisfy the equation (1.20) (p(−1) = −1 + 1 − 1 + 1 = 0), therefore one of the
root of equation (1.20) is −1. Thus x + 1 will be one of the factor of p(x). Also the given
equation is cubic, hence it will have three roots. Dividing the given polynomial by its factor
16 LESSON - 1. GENERAL PROPERTIES OF POLYNOMIAL EQUATIONS

we get the value of quotient. Rest two roots can be find by solving the equation of quotient.
To find the quotient we will use the synthetic division.

−1 1 1 1 1
-1 0 -1
1 0 1 0

Hence the quotient is given as q(x) = 1.x2 + 0.x + 1 = x2 + 1. Thus the root of equation
x2 + 1 = 0 are found as i, −i. Hence we can conclude that roots occurs in pair. Thus if
any polynomial whose all the coefficients are real and having one complex root then the
conjugate of that root will also exist.

Remark. If the coefficients given in equation (1.19) are complex instead of real then com-
plex root may or may not occurs in conjugate pair.

Example 1.13. Find the roots of the polynomial equation x2 − 7ix − 12 = 0.

Solution. Since the given equation is

p(x) = x2 − 7ix − 12 = 0. (1.21)

Solution of this equation is given as



7i ± −49 + 48
x=
2
7i ± i
x=
2
x =4i, 3i

Here 4i and 3i are not conjugate of each other because the coefficients of the polynomial
equation are not purely real.

Theorem 1.7. The irrational roots occurs in conjugate pair.

Or

In a given equation

a0 xn + a1 xn−1 + ... + an = 0 and a0 ̸= 0 (1.22)

where a0 , a1 , ...an are coefficients of the equation and the value √


of all the coefficients are
rational number. If √ the given polynomial has irrational root a + b then it will also has a
pair of its root a − b.

Proof. Proof of the theorem is omitted.

Example 1.14. Find the roots of the polynomial equation x2 − 4x + 1 = 0.


1.6. THEOREMS ON IMAGINARY, INTEGRAL AND RATIONAL ROOTS 17

Solution. The given equation is


p(x) = x2 − 4x + 1 = 0. (1.23)


4± 16 − 4
x=
2

4±2 3
x=
2√ √
x =2 + 3, 2 − 3.
√ √
Hence, we can conclude that roots occurs in pair (2+ 3, 2− 3). Thus, if any polynomial
whose all the coefficients are rational and having one irrational root then the conjugate of
that root will also be the roots of that polynomial equation.
Remark. If the coefficients given in equation (1.22) are irrational instead of real then irra-
tional root may or may not occurs in conjugate pair.

Example 1.15. Find the roots of the polynomial equation x2 − 3 3x + 6 = 0.
Solution. Since the given equation is

p(x) = x2 − 3 3x + 6 = 0. (1.24)
Solution of this equation is given as
√ √
3 3 ± 27 − 24
x=
√ 2

3 3± 3
x=
√ 2√
x =2 3, 3
√ √
Here 2 3 and 3 are not conjugate of each other because the coefficients of the polynomial
equation are not purely rational.
Theorem 1.8. Theorem on integral roots: Consider the equation of n degree
a0 xn + a1 xn−1 + a2 xn−2 ... + an = 0, (1.25)
whose all the coefficients a0 , a1 , a2 , ...an are integers, then any integer root of that equation
will be exact divisor of the constant term.
Proof. Let us assume that equation has an integer root ‘x’. Then from the above equation
(1.25)
−a0 xn − a1 xn−1 − a2 xn−2 ... = an , (1.26)
−x(a0 xn−1 + a1 xn−2 + a2 xn−3 ...) = an ,
Since all the coefficients of the given equation are integers. Hence the quantity in paren-
thesis will be integer and constant term on the right hand side is also integer. Therefore x
must be the complete or exact divisor of constant term.
18 LESSON - 1. GENERAL PROPERTIES OF POLYNOMIAL EQUATIONS

Remark. Converse of the above theorem may or may not be true i.e. any exact divisor
of constant term may or may not be the root of the equation. For example, consider an
equation 4x2 + 4x + 1 = 0. The exact divisor of constant term are ±1. But equation have
no integer roots i.e. both the roots are rational −1
2
, − 12 .

Example 1.16. Find all the integral roots of the equation

x3 + x2 − 3x + 9 = 0. (1.27)

Solution. Since all the coefficients of the given equation are integers, therefore first condi-
tion of the theorem is satisfied. Let us assume that equation has some integral roots. Then
they must divide the constant term. So the exact divisor of constant term are ±1, ±3, ±9.
Using synthetic division or factor theorem, we can check that ±1 are not the roots of the
equation.

−1 1 1 −3 9
−1 −1 4
1 0 −4 13

Since the last term of third row is not zero. Hence −1 is not root of this equation.
Similarly we can check for the other roots also.

−3 1 1 −3 9
−3 6 −9
1 −2 3 0

Hence −3 is the integral root of this equation. Rest two roots can be obtained by solving
the quotient equation i.e. 1.x2 − 2.x + 3 = x2 − 2x + 3 = 0

Example 1.17. Find all the integral roots of the equation x4 + 4x3 + 8x + 32 = 0.

Solution. Since the constant term here has numerous exact divisors, therefore finding the
roots will be quite difficult. To overcome this difficulty, we will transform this equation by
taking a transformation x = 2z into a equation whose constant term has less number of
divisor. The transformed equation become

(2z)4 + 4(2z)3 + 8(2z) + 32 = 0,


16z 4 + 32z 3 + 16z + 32 = 0,
z 4 + 2z 3 + z + 2 = 0. (1.28)

In the transformed equation (1.28) constant term is 2, whose divisors are ±1, ±2. Now
+1, +2 will not be the root of transformed equation (1.28) because all the coefficient here
are positive. Now we will check for −1 and −2.
1.6. THEOREMS ON IMAGINARY, INTEGRAL AND RATIONAL ROOTS 19

−1 1 2 0 1 2
−1 −1 1 −2
1 1 −1 2 0

−1 1 2 0 1 2
−1 −1 1 −2
−2 1 1 −1 2 0
−2 2 −2
1 −1 1 0

Hence the two integral roots of the transformed equation (1.28) are −1, −2 and the quotient
is z 2 −z+1 = 0. By solving the quotient by quadratic formula, we can find the rest two roots
of the equation (1.28). The integral roots of original equation are obtained by substituting
these values in the transformation x = 2z i.e. x = 2.(−1), x = 2.(−2). Thus the integral
roots of the original equation x4 + 4x3 + 8x + 32 = 0 are −2, −4.

1.6.1 Newton’s method for integral roots


From equation (1.26), in the previous theorem 1.8, we obtain that an must be divisible by x
or x must be complete divisor of an for a equation having integer coefficients. In equation
(1.26), by taking an and an−1 both on the right hand side, we can obtain one more condition

−a0 xn − a1 xn−1 − a2 xn−2 ... = an−1 x + an , (1.29)


−x2 (a0 xn−2 + a1 xn−3 + a2 xn−4 ...) = an−1 x + an ,
an
−x(a0 xn−1 + a1 xn−2 + a2 xn−3 ...) = an−1 + ,
x

in the similar way. Thus x2 must be divisor of an−1 x+an or x must be divisor of an−1 + axn .
Proceeding in the same way, we can obtained a set of conditions of divisibility which an
integral roots must satisfy. i.e. x must be divisor of an−2 + an−1x
+ axn2 . In last x must divide
the final sum a0 + ax1 + xa22 .... and the value of sum will be zero.

Remark. 1. x will be the root of any polynomial equation, if it satisfies all the condi-
tions of Newton’s method.

2. This method is quicker than synthetic division, as it detect the wrong choice at earlier
steps and throws it out.
a1 a2
3. The value of final sum a0 + x
+ x2
.... must be zero.

Example 1.18. Discuss the condition of Newton’s method for example 1.16 to check the
integral roots.
20 LESSON - 1. GENERAL PROPERTIES OF POLYNOMIAL EQUATIONS

Solution. Since in example 1.16 we check that −3 is the root of the given equation (1.27),
while 3 is not the root of the equation (1.27) using synthetic division. Here we will check
these roots by the conditions of Newton’s method. We will show that −3 satisfy all the
conditions of Newton’s method while 3 does not satisfies all.

−3/9 = −3
9
−3/ − 3 + =2
−3
−3 9
−3/1 + + = −1
−3 (−3)2
1 −3 9
−3/1 + + + =0
−3 (−3)2 (−3)3

Thus −3 divided all the values completely. Hence it satisfy all the conditions of Newton’s
method. Thus −3 will be the root of equation (1.27). On the other side

3/9 = 3
9
3/ − 3 + = 0
3
−3 9
3∤1+ +
3 (3)2

Since 3 does not satisfies all the conditions of Newton’s method. Hence 3 is not root of the
given equation. In the similar way, we can check the conditions for all the roots.

1.6.2 Rational Roots


If the rational number pq , a fraction in its lowest form (so that p and q are integers prime to
each other, and q ̸= 0 ) is the root of the given equation

a0 xn + a1 xn−1 + a2 xn−2 ... + an = 0, (1.30)

where all the coefficients a0 , a1 , a2 , ...an are integers and a0 ̸= 0, then p is the divisor of
an , while q is an divisor of a0 .

Remark. If in the equation (1.30), the value of coefficient a0 is 1, then all the rational roots
will be an integer.

Example 1.19. Find all the rational and integral roots of the equation

p(x) = 2x3 − 9x2 + 13x − 6 = 0 (1.31)

Solution. Here a0 = 2 and an = −6. Then for any rational root of the form rs r must be
divisor of −6 while s must be divisor of 2. Since all the divisor of 2 are ±1, ±2 and all the
divisor of −6 are ±1, ±2, ±3, ±6. Hence all the possible number of fractional roots of the
1.7. DESCARTES’ RULE OF SIGNS 21

±1 ±3
given equation (1.31) are ±2 , ±2 ie 12 , − 12 , 32 , − 23 . By substituting all the values one by one
in equation (1.31), we can find the rational roots of the given equation.
   3  2  
1 1 1 1 3
p =2 −9 + 13 − 6 = − ̸= 0
2 2 2 2 2
   3  2  
1 1 1 1
p − =2 − −9 − + 13 − − 6 = −15 ̸= 0
2 2 2 2
   3  2  
3 3 3 3
p =2 −9 + 13 −6=0
2 2 2 2
   3  2  
3 3 3 3 210
p − =2 − −9 − + 13 − −6=− ̸= 0
2 2 2 2 4

3
Thus, one integral root of the given equation is 2
and rest roots can be found by using
integral theorem 1.8 and Newton’s method 1.6.1.
Remark. Some important results on roots of an equation
1. Every equation of an odd degree has at least one real roots of a sign opposite to that
of its last term.

2. Every equation of an even degree, whose last term is negative, has at least two real
roots, one positive and the other one negative.
In-text Exercise 1.3. Find all the integral and rational roots of the following equations
(a) x3 + 8x2 + 13x + 6 = 0

(b) x3 + 12x2 − 32x − 256 = 0

(c) 2x3 + 3x2 − 11x − 6 = 0

(d) 32x3 − 6x − 1 = 0.

(e) Find the equation of the lowest degree with real coefficients having 1 + i and 2 − i as
two of its roots

1.7 Descartes’ rule of signs


1.7.1 Descartes’ rule of signs-Positive roots
In algebra, Descartes’ rule of signs is used for finding the maximum possible number of
positive real roots of a polynomial without actually solving it. It state that
The number of positive root of an equation p(x) = 0 can not exceed the number of changes
of sign from (+ to −) or from (− to +) in the terms occurring in p(x).
Or
22 LESSON - 1. GENERAL PROPERTIES OF POLYNOMIAL EQUATIONS

The possible number of the positive root of an equation p(x) = 0 is equal to the number of
sign changes in the coefficients of the terms or less than the sign changes by a multiple of
2.
Remark. 1. This rule does not give the exact number of roots of the polynomial. Also,
it does not identify the roots of the polynomial.
2. Before applying the Descartes’ rule of signs make sure to arrange the terms of the
polynomial in descending order. For example it should be in the order ..., x5 , x4 , x3 ,
x2 , x and constant term.
3. While counting the sign change, do not write the terms that have a coefficient to be
0. For example 3x2 − 1 will not be written as 3x2 + 0x − 1.
Example 1.20. Find the possible number of positive real roots of the polynomial x3 +3x2 −
x − x4 − 2.
Solution. Since the terms of the polynomial are not in the descending order of exponents,
therefore we will first make the terms in descending order of exponent.

p(x) = −x4 + x3 + 3x2 − x − 2 (1.32)

Now, we will count the number of sign changes in the given polynomial (1.32).

from − x4 to + x3 there is one sign change(−to+)


from + x3 to + 3x2 there is no sign change(+to+)
from + 3x2 to − x there is one sign change(+to−)
from − x to − 2 there is no sign change(−to−)

Thus there are two sign changes in the given polynomial p(x) and hence possible number
of positive real roots of the polynomial is 2 or 0.

1.7.2 Descartes’ rule of signs-Negative roots


In a polynomial equation p(x) = 0, if x is replaced by −x, then the resulting equation
will have the same roots as the original except that their signs will be changed. Thus the
negative roots of p(x) = 0 are the positive roots of p(−x) = 0. Hence the Descartes’s rule
of signs for negative roots is stated as
The number of negative real roots of an equation p(x) = 0 can not exceed the number of
changes of sign from (+ to −) or from (− to +) in the terms occurring in p(−x).
Or
The possible number of the negative roots of the polynomial p(x) is equal to the number
of sign changes in the coefficients of the terms of p(−x) or less than the sign changes by a
multiple of 2.
Remark. 1. This rule does not give the exact number of roots of the polynomial. Also,
it does not identify the roots of the polynomial.
1.7. DESCARTES’ RULE OF SIGNS 23

2. Before applying the Descartes’ rule of signs make sure to arrange the terms of the
polynomial in descending order of exponents.

Example 1.21. Find the possible number of real roots of the polynomial x3 −x2 −14x+24
and verify.

Solution. Since the terms of the polynomial are already in the descending order of expo-
nents, therefore we will count the number of sign changes in the given polynomial

p(x) = +x3 − x2 − 14x + 24. (1.33)

from + x3 to − x2 there is one sign change(+to−)


from − x2 to − 14x there is no sign change(−to−)
from − 14x to + 24 there is one sign change(−to+)

Thus there are two sign changes in the given polynomial p(x) and hence possible number of
positive real roots of the polynomial is 2 or 0. Similarly for finding the number of negative
roots, we will check the sign change in the given polynomial by replacing x by −x. Thus
by taking (−x) in place of x, required polynomial becomes

p(−x) =(−x)3 − (−x)2 − 14(−x) + 24


= − x3 − x2 + 14x + 24 (1.34)

from − x3 to − x2 there is no sign change(−to−)


from − x2 to + 14x there is one sign change(−to+)
from + 14x to + 24 there is no sign change(+to+)

Thus there are one sign changes in the given polynomial p(−x) and hence possible number
of negative real roots of the polynomial p(x) is 1.

Verification: Since the polynomial given in equation (1.33) is cubic, therefore we can find
the roots of this equation. The roots of the equation are obtained as 2, 3, −4. Thus we can
see that there are two positive roots and one negative root of the given polynomial (1.33).

Example 1.22. Determine the possible number of real solutions of the polynomial 4x7 +
3x6 + x5 + 2x4 − x3 + 9x2 + x + 1 = 0.

Solution. For the possible positive roots, we will check the sign changes in the given poly-
nomial

p(x) = +4x7 + 3x6 + x5 + 2x4 − x3 + 9x2 + x + 1. (1.35)

from + 2x4 to − x3 there is one sign change(+to−)


from − x3 to + 9x2 there is one sign change(−to+)
24 LESSON - 1. GENERAL PROPERTIES OF POLYNOMIAL EQUATIONS

Thus there are two sign changes in the given polynomial p(x) and hence possible number
of positive real solution of the polynomial is 2 or 0. For finding the possible number of
negative solution, we will check the sign change in the given polynomial by replacing x by
−x. Thus by taking (−x) in place of x, required polynomial becomes

p(−x) = + 4(−x)7 + 3(−x)6 + (−x)5 + 2(−x)4 − (−x)3 + 9(−x)2 + (−x) + 1.


= − 4x7 + 3x6 − x5 + 2x4 + x3 + 9x2 − x + 1 (1.36)

from − 4x7 to + 3x6 there is one sign change(−to+)


from + 3x6 to − x5 there is one sign change(+to−)
from − x5 to + 2x4 there is one sign change(−to+)
from + 9x2 to − x there is one sign change(+to−)
from − x to + 1 there is one sign change(−to+)

Since there are five sign changes in the given polynomial p(−x) therefore possible number
of negative real solutions of the polynomial p(x) is 5, 3 or 1.

Thus there are two or zero positive solutions, and five, three or one negative solutions of
given polynomial (1.35).

1.7.3 Application of Descartes’ rule of signs for finding the Imaginary


roots
In a polynomial equation p(x) = 0, if the sum of number of maximum possible positive
roots and maximum possible negative roots are less then the degree of the polynomial then
there will be existence of imaginary roots in that polynomial. It is also clear that for the
real coefficients, the imaginary roots occur in conjugate pair.
The number of imaginary roots in a given polynomial equation p(x) = 0 is either equal to
(n − p − q) or greater then (n − p − q) by an even number where n is the degree of the
polynomial while p and q are the positive and negative roots of the polynomial respectively.

Remark. This application of Descartes’s rule of signs for finding the imaginary roots is
possible only in case of incomplete equation. For the complete equation total number of
sign variation in p(x) and p(−x) will be equal to degree of the polynomial.(A polynomial
is called incomplete if the coefficient of some terms are zero. For example x4 + x − 1 is
incomplete polynomial as the coefficient of x3 and x2 term is zero while x4 −x3 +x2 −x+1
is a complete polynomial.)

Example 1.23. Prove that the equation 3x7 − x4 + x3 − 1 = 0, has at least four imaginary
roots.

Solution. Let the equation be p(x) = 0, where

p(x) = +3x7 − x4 + x3 − 1. (1.37)


1.8. SUMMARY 25

The number of sign change in p(x) is


from + 3x7 to − x4 there is one sign change(+to−)
from − x4 to + x3 there is one sign change(−to+)
from + x3 to − 1 there is one sign change(+to−)
Thus there are three changes of sign and hence possible number of positive real roots of the
polynomial are 3 or 1. The number of sign change in p(−x) is
p(−x) = − 3x7 − x4 − x3 − 1
Since all the terms of p(−x) are negative, hence there is no sign change for p(−x). Thus
there is no real negative roots of the given polynomial. Now we will construct a table with
all possibilities. Note that the degree of the given polynomial (1.37) is 7.

Number of Number of Number of


positive real negative real imaginary
roots roots roots
3 0 7−(3+0) = 4
1 0 7−(1+0) = 6

Hence the total number of imaginary roots of equation (1.37) are either 4 or 6.
In-text Exercise 1.4. Solve the following questions:
(a) Find the possible number of positive, negative and imaginary roots of the polynomial
equation x3 − x2 + x − 1.
(b) Find the nature of the roots of the equation x4 + 15x2 + 7x − 11 = 0.
(c) Show that the equation xn + 1 = 0, has no real roots when n is even and −1 is the
only root when n is odd.
(d) Find the number of imaginary roots of the equation x4 − 3x2 − x + 1 = 0

1.8 Summary
In the end of the chapter, we know
1. Polynomials can be written in the factored form.
2. Using synthetic division, any polynomial can be divided by a linear polynomial with-
out performing a long calculations.
3. Imaginary and irrational roots occurs in pair for the real and rational coefficients
respectively.
4. How to find the number of positive, negative and imaginary roots of an polynomial
equation without solving it.
26 LESSON - 1. GENERAL PROPERTIES OF POLYNOMIAL EQUATIONS

1.9 Self-Assessment Exercises


1. Check whether the polynomial p(x) = 4x3 + 4x2 − x − 1 is a multiple of 2x + 1.

2. For what value of k is the polynomial p(x) = 2x3 − kx2 + 3x + 10 exactly divisible
by x − 2.

3. If two polynomials 2x3 +ax2 +4x−12 and x3 +x2 −2x+a leave the same remainder
when divided by x − 3, find the value of a and also find the remainder.

4. Find the quotient of x3 − 5x2 − 2x + 24 by x − 4, and then divide the quotient by


x − 3. What are the roots of x3 − 5x2 − 2x + 24 = 0.

5. If two roots of the equation x4 − 2x3 − 12x2 + 10x + 3 = 0 are 1 and −3, then find
the remaining two roots.

6. Find√the equation of the lowest degree with rational coefficients having 3 − 5 and
5 + 2 as two of its roots.

7. Find all the integral and rational roots of 6y 3 − 11y 2 + 6y − 1 = 0.

8. Find all the integral roots of x4 + 4x3 + 8x + 32 = 0 by using Newton’s method for
integral roots.

9. Find all the rational roots of x3 − 12 x2 − 2x + 1 = 0.

10. Find the nature of roots of the given equation

xn + 1 = 0

when n is either odd or even.

11. Find all possible values of imaginary roots for the given polynomial p(x) = x3 +
3x2 − x − x5 + 7.

12. Find the superior limit of the number of imaginary roots of the equation

5x8 − 6x3 + x2 + 1 = 0.

1.10 Solutions to In-text Exercises


Exercise 1.1
(a) By substituting x = 3 in polynomial p(x) = x3 − 3x2 + 6x − 5, we get p(3) = 13,
which is the value of remainder.

(b) same as above

(c) By substituting x = − 12 in p(x) = 4x3 − 2x2 + kx + 5, we get 4 − k


2
which is given
to be equal to −10. Thus k = 28.
1.10. SOLUTIONS TO IN-TEXT EXERCISES 27

(d) By substituting x = −1 in polynomial p(x) = 18x10 + 9x5 + 1, we get p(−1) = 0.


Exercise 1.2
(a) This can be done by synthetic division as

3 1 6 10 −1
3 27 111
1 9 37 110

the remainder is 110 and the value of quotient is 1.x2 + 9.x + 37 = x2 + 9x + 37.

(b) This can be done by synthetic division as


5
3
3 −17 −1 15
5 −20 −35
3 −12 −21 −20

the remainder is −20 and the value of quotient is 3.x2 − 12.x − 21 = 3x2 − 12x − 21.

(c) x2 − 4 = 0 is (x − 2)(x + 2) = 0

2 2 −1 −6 4 −8
4 6 0 8
2 3 0 4 0

and

−2 2 3 0 4
−4 2 −4
2 −1 2 0

The value of quotient is 2.x2 − 1.x + 2 = 2x2 − x + 2.


Exercise 1.3
(a) Since x3 + 8x2 + 13x + 6 has the coefficient of highest degree as 1. Therefore all the
rational roots will be integers. Also factor/divisor of 6 are ±1, ±2, ±3, ±6. Now all
the terms of the given equation are positive. Thus there will be no positive roots of
the given equation. Hence total choices of roots are −1, −2, −3, −6. Using Newton’s
method we can check that −2 & − 3 does not satisfy Newton’s condition. Thus all
the roots of given equation are −1, −1, −6, where −1 is the root with multiplicity 2.
28 LESSON - 1. GENERAL PROPERTIES OF POLYNOMIAL EQUATIONS

(b) Since the constant term here has numerous exact divisors. Therefore we will trans-
form this equation into a simpler equation by taking a transformation x = 4z.

(c) All possible roots of the given equation 2x3 + 3x2 − 11x − 6 are ±1, ±2, ±3, ±6,
±1 ±3
, . Using Newton’s method we can check that 2 & − 3 satisfy Newton’s all
±2 ±2
condition. Also p − 12 = 0. Thus, all the roots of given equation are 2, −3, − 12 .


(d) All possible roots of the given equation 32x3 − 6x − 1 are ±1, ±1 , ±1 , ±1 , ±1 , ±1 .
±2 ±4 ±8 ±16 ±32

(e) Since the coefficient of the required equation are given to be real. Therefore in that
case complex roots occurs in pair. Thus the required equation must have at least four
roots as 1 + i, 1 − i, 2 − i and 2 + i. The equation is

(x − (1 − i))(x − (1 + i))(x − (2 + i))(x − (2 − i)) = 0


((x − 1) + i)((x − 1) − i))((x − 2) − i)((x − 2) + i) = 0
((x − 1)2 + 1)((x − 2)2 + 1) = 0.

is the required equation.

Exercise 1.4

(a) Since total number of sign change in p(x) = 0 are 3. Hence total number of positive
roots are 3, 1. Also, total number of sign change in p(−x) = 0 are 0. Hence total
number of negative roots are 0.

Number of Number of Number of


positive real negative real imaginary
roots roots roots
3 0 3−(3+0) = 0
1 0 3−(1+0) = 2

Hence the total number of imaginary roots of given polynomial equation are either 0
or 2.

(b) Since there are one sign change in case of p(x) and one sign change in case of p(−x).
Thus there are one positive and one negative roots and 4 − (1 + 1) = 2 imaginary
roots.

(c) When n is even then there is no sign change in p(x) and p(−x). Hence no real root
with exist. Also in case of odd power of n, there is no sign change in p(x), while one
sign change in case of p(−x). Thus only one negative root will exist which will be
−1.

(d) Since there are two sign change in p(x) and two sign change in p(−x). Hence
1.10. SOLUTIONS TO IN-TEXT EXERCISES 29

Number of Number of Number of


positive real negative real imaginary
roots roots roots
2 2 4−(2+2) = 0
0 0 4−(0+0) = 4
2 0 4−(2+0) = 2
0 2 4−(0+2) = 2

The inferior limit to the imaginary roots of given polynomial equation is 0.

Suggested Readings
1. Burnside, W.S., & Panton, A.W. (1979). The Theory of Equations. Vol. 1. Eleventh
Edition, (Fourth Indian Reprint. S. Chand & Co. New Delhi), Dover Publications,
Inc.

2. Dickson, Leonard Eugene (2009). First Course in the Theory of Equations. John Wi-
ley & Sons, Inc. The Project Gutenberg eBook (http://www.gutenberg.org/ebooks/29785).
Lesson - 2

Relation Between the Roots and


Coefficients of Equations

Structure
2.1 Learning Objectives 30
2.2 Introduction 30
2.3 Relation Between Roots and Coefficients of Equations 31
2.4 Applications to solution of Equations 36
2.5 Summary 44
2.6 Self Assessment Exercise 45
2.7 Solutions to In-text Exercises 46

2.1 Learning Objectives


Students will be able to
• recognize the relation between the roots and coefficients of a quadratic and cubic
equation.

• understand how the relation between the roots and coefficients of a quadratic and
cubic equation can be extended to n degree polynomial equation.

• find the roots of a higher order polynomial equation when relation between its roots
are given.

2.2 Introduction
Roots of polynomials are solutions for given polynomials where the function is equal to
zero. When its comes to polynomials, roots becomes particularly important. They allow us
to break down our polynomial equation into simpler terms that we can understand and solve

30
2.3. RELATION BETWEEN ROOTS AND COEFFICIENTS OF EQUATIONS 31

more easily. In the previous chapter, we had learned a lot of theorems based on roots of the
polynomials. In this chapter, we will learn the relation between roots and coefficients of a
polynomial equation. Using this relation, we will be able to find the roots of a polynomial
equation of degree n.
A polynomial equation of degree n
p(x) ≡ a0 xn + a1 xn−1 + a2 xn−2 ...an−1 x + an = 0, (2.1)
has exactly n roots. Let the n roots of equation (2.1) are α1 , α2 , ..., αn , then
p(x) ≡ a0 (x − α1 )(x − α2 )...(x − αn ). (2.2)

2.3 Relation Between Roots and Coefficients of Equations


Here we will learn relation between roots and coefficients of a quadratic and cubic equation
first. Later this result will be extended for any polynomial of degree n.

Relation between roots and coefficients of an quadratic equation


Consider a quadratic equation
ax2 + bx + c = 0, (2.3)
whose roots are given as α1 and α2 . Roots of the quadratic equations (2.3) can be found
using quadratic formula

−b ± b2 − 4ac
x= . (2.4)
2a
√ √
−b+ b2 −4ac −b− b2 −4ac
Therefore, the roots of equation (2.3) are found as α1 = 2a
, α2 = 2a
.

Now, the sum of the roots is


√ √
−b + b2 − 4ac −b − b2 − 4ac
α1 + α2 = +
2a 2a
2b b coefficient ofx
=− =− =− .
2a a coefficient ofx2
and the product of the roots is
 √   √ 
−b + b2 − 4ac −b − b2 − 4ac
α1 .α2 = .
2a 2a
 2  √ 2 2
b b − 4ac
= − −
2a 2a
4ac
=
(2a)2
c constant term
= = .
a coefficient ofx2
32LESSON - 2. RELATION BETWEEN THE ROOTS AND COEFFICIENTS OF EQUATIONS

coefficient ofx constant term


Thus α1 + α2 = − coefficient ofx2
and α1 .α2 = coefficient ofx2
represent the required relations
between roots (i.e. α1 and α2 ) and coefficients (i.e. a, b and c) of the equation ax2 +
bx + c = 0.
Example 2.1. Find the zeros/roots of the polynomial p(x) = x2 + 7x + 12 and verify the
relation between its zeros and coefficients.
Solution. The roots of the given equation are obtained as
x2 + 7x + 12 = 0, (2.5)
x2 + 3x + 4x + 12 = 0,
(x + 3)(x + 4) = 0,
x = −3, −4.
Thus, the roots of the equation (2.5) are α1 = −3, α2 = −4.

Verification:
coefficient ofx 7
Sum of the roots (α1 + α2 ) = −3 + (−4) = − 2
= − = −7
coefficient ofx 1
constant term 12
Product of the roots (α1 .α2 ) = −3.(−4) = = = 12.
coefficient ofx2 1
Example 2.2. If α and β are the roots of the equation 3x2 − 5x + 2 = 0, then find the value
of
1. α2 + β 2
2. α3 + β 3
1 1
3. α
+ β

Solution. The given equation is


3x2 − 5x + 2 = 0. (2.6)
Since, the roots of equation (2.6) are α and β, therefore using the relation between roots
and coefficients, we get
 
coefficient ofx −5 5
α+β =− = − =
coefficient ofx2 3 3
contant term 2
α.β = 2
= (2.7)
coefficient ofx 3
1. Now
α2 + β 2 = (α + β)2 − 2αβ
 2  
5 2
= − 2.
3 3
13
= .
9
2.3. RELATION BETWEEN ROOTS AND COEFFICIENTS OF EQUATIONS 33

2. We know

α3 + β 3 = (α + β)3 − 3αβ(α + β)
 3   
5 2 5
= − 3.
3 3 3
35
= .
27
3. Also,
1 1 (α + β)
+ =
α β αβ
 
5/3
=
2/3
5
= .
2

Relation between roots and coefficients of an cubic equation


Consider a cubic equation

ax3 + bx2 + cx + d = 0, (a ̸= 0) (2.8)

whose roots are given as α1 , α2 and α3 . Therefore, by equation (2.1) and (2.2)

ax3 + bx2 + cx + d ≡a(x − α1 )(x − α2 )(x − α3 )


b c d
x3 + x2 + x + ≡(x − α1 )(x − α2 )(x − α3 )
a a a
b c d
x3 + x2 + x + ≡x3 − (α1 + α2 + α3 )x2 + (α1 α2 + α2 α3 + α1 α3 )x − α1 α2 α3 .
a a a
On comparing the same power’s of x, we get the following relations
3
X b coefficient ofx2
α1 + α2 + α3 = αi = − =−
i=1
a coefficient ofx3
3
X c coefficient ofx
α1 α2 + α2 α3 + α1 α3 = αi αj = =
i,j=1
a coefficient ofx3
3
Y d constant term
α1 α2 α3 = αi = − =−
i=1
a coefficient ofx3
2
coefficient ofx coefficient of x
Thus α1 + α2 + α3 = − coefficient ofx3
, α1 .α2 + α2 α3 + α1 α3 = coefficient ofx3
, and α1 .α2 .α3 =
constant term
− coefficient ofx3
represent the required relations between roots (i.e. α 1 α2 and α3 ) and
,
3 2
coefficients (i.e. a, b, c and d) of the equation ax + bx + cx + d = 0.
Example 2.3. Find the equation whose roots are given as 3, 3, −2. Also, verify the relation
between roots and coefficients.
34LESSON - 2. RELATION BETWEEN THE ROOTS AND COEFFICIENTS OF EQUATIONS

Solution. Given three roots are α1 = 3, α2 = 3, α3 = −2. The required equation is given
as

(x − α1 )(x − α2 )(x − α3 ) = 0
(x − 3)(x − 3)(x − (−2)) = 0
x3 − 4x2 − 3x + 18 = 0 (2.9)

Verification:

coefficient of x2
 
−4
Sum of the roots (α1 + α2 + α3 ) = 3 + 3 − 2 = − 3
=− = 4.
coefficient of x 1
Sum of the product of two roots (α1 α2 + α1 α3 + α2 α3 ) = 3.3 + 3.(−2) + 3.(−2)
 
coefficient of x −3
= 3
= = −3.
coefficient of x 1
 
constant term 18
Product of the roots (α1 .α2 .α3 ) = 3.3.(−2) = − 3
=− = −18.
coefficient of x 1

In-text Exercise 2.1. Solve the following questions:

(a) If the difference between roots of the equation x2 − 13x + k = 0 is 17. Find k

(b) If α and β are the roots of the equation 3x2 + 7x − 2 = 0, find the values of

α β
(i) β
+ α

α2 β2
(ii) β
+ α

Relation Between Roots and Coefficients of n degree equation


Consider a polynomial equation of degree n

a0 xn + a1 xn−1 + ... + an−1 x + an = 0, (2.10)

whose roots are given by α1 , α2 , α3 ..., αn . Therefore by using equation (2.1) and (2.2)

a0 xn + a1 xn−1 + a2 xn−2 + ... + an ≡ a0 (x − α1 )(x − α2 )(x − α3 )...(x − αn )


a1 a2 an
xn + xn−1 + xn−2 + ... + ≡ (x − α1 )(x − α2 )(x − α3 )...(x − αn )
a0 a0 a0
a1 a2 an
xn + xn−1 + xn−2 + ... + ≡ x2 − (α1 + α2 )x + α1 α2 (x − α3 )...(x − αn )

a0 a0 a0
a 1 a 2 a n
xn + xn−1 + xn−2 + ... + ≡ xn − (α1 + α2 + α3 + ... + αn )xn−1
a0 a0 a0
+(α1 α2 + α1 α3 + ... + α2 α3 + ...)xn−2 + ...(−1)n α1 α2 α3 ...αn .
2.3. RELATION BETWEEN ROOTS AND COEFFICIENTS OF EQUATIONS 35

On comparing the same power’s of x, we get the following relations


n
X a1 coefficient of xn−1
S1 = α1 + α2 + α3 + ...αn = αi = − =−
i=1
a0 coefficient of xn
n
X a2 coefficient of xn−2
S2 = α1 α2 + α1 α3 + ... + α2 α1 + ... = αi αj = =
i,j=1
a0 coefficient of xn
..
.
n
Y an constant term
Sn = α1 α2 α3 ...αn = αi =(−1)n = (−1)n
i=1
a0 coefficient of xn

Here, Sk denotes the sum of the products of the roots taken k at a time. For example S2
denotes the sum of the product of the roots taken 2 at a time.

Remark. If in the equation (2.10), coefficient of the highest term is unity, i.e.

xn + a1 xn−1 + ... + an−1 x + an = 0, (2.11)

then, the above relation become quite easy to apply. In that case sum of the roots become
negative times of coefficient of xn−1 while sum of the product of the two roots becomes
coefficient of xn−2 and same as follows with alternate sign change. i.e.

S1 = α1 + α2 + α3 + ...αn = − coefficient of xn−1 = −a1


S2 = α1 α2 + α1 α3 + ... + α2 α1 + ... = coefficient of xn−2 = a2
..
.
Sn = α1 α2 α3 ...αn = (−1)n constant term = (−1)n an

Remark. Some special roots of a cubic equation

1. If three roots of a cubic equation are given in A.P. (arithmetic progression), then roots
may be chosen as a − d, a, a + d. The choice of particular form of these roots make
the calculation easier to solve. As in this case

Sum of the roots = a − d + a + a + d = 3a


Sum of product of the two roots = (a − d).a + a.(a + d) + (a + d).(a − d)
= a2 − ad + a2 + ad + a2 − d2
= 3a2 − d2
product of the roots = (a − d).a.(a + d)
= a(a2 − d2 ).

2. If three roots of a cubic equation are given in G.P. (geometric progression), then roots
36LESSON - 2. RELATION BETWEEN THE ROOTS AND COEFFICIENTS OF EQUATIONS

may be chosen as ar , a, ar. In this case


a
Sum of the roots = + a + ar
r
a a
Sum of product of the two roots = .a + a.ar + ar.
r r
a2
= + a2 r + a2
r
a
product of the roots = .a.ar
r
= a3

3. If three roots of a cubic equation are given in H.P. (harmonic progression), then roots
2αγ
may be chosen as α, β, γ where β = α+γ .
Note: A harmonic progression is a progression which are formed by taking the re-
ciprocals of an arithmetic progression i.e. if α, β, γ are in H.P. then α1 , β1 , γ1 will be in
A.P. Therefore, we have
1 1 1 1
− = −
β α γ β
2 α+γ
=
β αγ
2αγ
β=
α+γ

Remark. Some special roots of a bi-quadratic equation


1. If four roots of a bi-quadratic equation are given in A.P. (arithmetic progression),
then roots may be chosen as a − 3d, a − d, a + d, a + 3d.
2. If four roots of a bi-quadratic equation are given in G.P. (geometric progression), then
roots may be chosen as ra3 , ar , ar, ar3 .
3. If four roots of a bi-quadratic equation are given in H.P. (harmonic progression), then
2αγ 2βδ
roots may be chosen as α, β, γ, δ where β = α+γ , γ = β+δ .

2.4 Applications to solution of Equations


We are sometimes required to find conditions under which the roots of given equations are
related. Also, sometimes we have to solve a equation when a relation between its roots are
given or roots are related by some relation. Following examples will illustrate the procedure
for solving such types of problems.
Example 2.4. Solve the equation

x3 − 9x2 + 23x − 16 = 0, (2.12)

whose roots are in Arithmetic progression (A.P.)


2.4. APPLICATIONS TO SOLUTION OF EQUATIONS 37

Solution. Let the roots be chosen as a − d, a, a + d. Therefore, we get


coefficient ofx2
Sum of the roots = a − d + a + a + d = 3a = − =9
coefficient ofx3
3a = 9
a = 3.
coefficient ofx
Also (a − d).a + a.(a + d) + (a − d).(a + d) = 3a2 − d2 = = 23
coefficient ofx3
3a2 − d2 = 23
3.(3)2 − d2 = 23
d2 = 4
d = ±2
Thus by taking a = 3, d = 2, roots become a − d = 1, a = 3, a + d = 5 i.e. 1, 3, 5.

Note: By taking a = 3, d = −2, we get the same set of roots as a − d = 5, a = 3, a + d = 1


i.e. 5, 3, 1.
Example 2.5. Solve the equation
2x3 − x2 − 22x − 24 = 0 (2.13)
two of its roots being in the ration 3 : 4.
Solution. Since the roots are given in the ratio 3 : 4, therefore let us assume the roots of
the equation (2.13) as 3α, 4α, β. Thus we have
1
3α + 4α + β =
2
1
7α + β = (2.14)
2
22
3α.4α + 4α.β + 3α.β = − = −11
2
2
12α + 7αβ = −11 (2.15)
 
−24
3α.4α.β = − = 12
2
12α2 β = 12
α2 β = 1 (2.16)
From eqution (2.14) and (2.15), we get
 
2 1
12α + 7α − 7α = −11
2
−74α2 + 7α + 22 = 0

7 ± 49 + 6512
α=
2.(74)
1 22
α=− , . (2.17)
2 37
38LESSON - 2. RELATION BETWEEN THE ROOTS AND COEFFICIENTS OF EQUATIONS

Since α = 22 37
does not satisfy the equation (2.13). Hence by putting the value of α = − 21
in equation (2.16), the value of β = 4. Thus three roots of equation (2.13) are given as
− 32 , −2, 4.
Example 2.6. Solve the equation

3x3 − 26x2 + 52x − 24 = 0, (2.18)

whose roots are in G.P.


Solution. Let the roots in G.P. are ar , a, ar. We know,
 
a −26 26
+ a + ar = − = (2.19)
r 3 3
a a 52
.a + a.ar + .ar =
r r 3
a2 52
+ a2 r + a2 = (2.20)
r 3 
a −24
.a.ar = − =8
r 3
a3 = 8 (2.21)

Dividing equation (2.19) by equation (2.20), we get

a 26
r
+ a + ar 3
a = 52
a( r + ar + a) 3
1 1
=
a 2
a = 2.

Putting the value of ‘a’ in equation (2.19), we get


2 26
+ 2 + 2r =
r 3
6r2 − 20r + 6 = 0
3r2 − 10r + 3 = 0
3r2 − 9r − r + 3 = 0
(3r − 1)(r − 3) = 0
1
r = ,3
3
Taking a = 2 and r = 3, roots of the equation (2.18) are obtained as 23 , 2, 6.

Note:- By taking a = 2 and r = 13 , we get the same sets of roots.


Example 2.7. Solve the equation x4 + 2x3 − 21x2 − 22x + 40 = 0, whose roots are given
in A.P.
2.4. APPLICATIONS TO SOLUTION OF EQUATIONS 39

Solution. Let the roots be chosen as a − 3d, a − d, a + d, a + 3d. Then by the relation
between roots and coefficients, we get
a − 3d + a − d + a + d + a + 3d = −2
4a = −2
1
a=− .
2
Also,
(a − 3d)(a − d) + (a − d)(a + d) + (a + d)(a + 3d) + (a + 3d)(a − 3d) + (a − 3d)(a + d)
+(a + 3d)(a − d) = −21
6a2 − 10d2 = −21
 2   
−1 2 1
6 + 21 = 10d ∵ a=−
2 2
9
d2 =
4
3
d=± .
2
1 3
Taking a = − 2 and d = 2 , four roots of given equation are obtained as
   
1 3
a − 3d = − −3 = −5
2 2
   
1 3
a−d=− − = −2
2 2
   
1 3
a+d=− + =1
2 2
   
1 3
a + 3d = − +3 =4
2 2
Thus the four roots of given equation are −5, −2, 1, 4.

Note: By taking a = − 12 and d = − 32 , we will get the same set of roots.


Example 2.8. Solve the cubic equation
2x3 − 9x2 + 12x − 4 = 0, (2.22)
given that two of its roots are equal.
Solution. Let the three roots of the equation are α, α, β (since two roots are equal). Then
we have
9
α + α + β = 2α + β = (2.23)
2
12
α.α + α.β + β.α = α2 + 2αβ = =6 (2.24)
 2
−4
α.α.β = α2 β = − =2 (2.25)
2
40LESSON - 2. RELATION BETWEEN THE ROOTS AND COEFFICIENTS OF EQUATIONS

9
From equation (2.23), β = 2
− 2α. Putting the value of β in equation (2.24), we get
 
2 9
α + 2α − 2α = 6
2
−3α2 + 9α − 6 = 0
α2 − 3α + 2 = 0
α = 1, 2.

Since α = 1 does not satisfy the given equation (2.22). Therefore by taking α = 2, we get
β = 21 . Thus, three roots of the given equation (2.22) are found as 2, 2, 12 .
Example 2.9. Solve the equation

3x3 + 11x2 + 12x + 4 = 0, (2.26)

whose roots are given in Harmonic progression (H.P.)


Solution. Let the three roots in harmonic progression of the given equation are α, β, γ
2αγ
where β = α+γ . Since

11
α+β+γ =− (2.27)
3
12
αβ + βγ + γα = =4 (2.28)
3
4
αβγ = − . (2.29)
3
Also,
2αγ
β=
α+γ
αβ + βγ = 2αγ
αβ + βγ + αγ = 2αγ + αγ = 3αγ (2.30)

From equation (2.28) and (2.30), we get

3αγ = 4
4
αγ = .
3
Putting the value of αγ in equation (2.29), we get
4 4
.β = −
3 3
β = −1.

Thus, one of the root of the equation (2.26) is obtained as β = −1. Therefore, by factor
theorem 1.2 (x + 1) will be one of the factor of equation (2.26). Thus dividing the equation
(2.26) by (x + 1), we get
2.4. APPLICATIONS TO SOLUTION OF EQUATIONS 41

−1 3 11 12 4
−3 −8 −4
3 8 4 0

Hence the reduced quadratic equation is obtained as 3x2 + 8x + 4 = 0. By solving this


equation we get

−8 ± 64 − 48 −8 ± 4 −2
x= = = , −2. (2.31)
6 6 3
Thus all the required roots of the equation (2.26) are −1, −2, −2/3.
Example 2.10. Find the necessary condition for the roots of the equation
x3 − px2 + qx − r = 0, (2.32)
to be in
(i) A.P.
(i) G.P.
(i) H.P.
Solution. To find the necessary condition for the roots to be in A.P., G.P., and H.P., we have
to find a relation between coefficients of the equation by using relation between the roots.
(i) Let the roots in A.P. are a − d, a, a + d, then we know
a − d + a + a + d = −(−p) = p
3a = p
p
a= (2.33)
3
(a − d).a + a.(a + d) + (a − d)(a + d) = q
3a2 − d2 = q (2.34)
(a − d).a.(a + d) = r
a3 − ad2 = r (2.35)
Since a is one of the root of equation (2.32). Therefore it will satisfy the equation.
Hence we get
a3 − pa2 + qa − r = 0. (2.36)
Putting the value of a from equation (2.33) in equation (2.36), we get
 p 3  p 2 p
−p +q −r =0
3 3   33   3 
p p pq
− + −r =0
27 9 3
2p3 − 9pq + 27r = 0 (2.37)
42LESSON - 2. RELATION BETWEEN THE ROOTS AND COEFFICIENTS OF EQUATIONS

is the required relation/condition for the roots to be in A.P.

Note: This relation can also be found by solving equation (2.33) and (2.34) for a and
d and putting the value of a and d in equation (2.35).

(i) Let the roots in G.P. are ad , a, ad, then we know


a
+ a + ad = −(−p) = p (2.38)
d
a a
.a + a.(ad) + (ad) = q
d d
2
a
+ a2 d + a2 = q (2.39)
d
a
.a.(ad) = r
d
a3 = r
1
a = (r) 3 (2.40)

Since a is one of the root of equation (2.32). Therefore it will satisfy the equation.
Hence, we get

a3 − pa2 + qa − r = 0. (2.41)

Putting the value of a from equation (2.40) in equation (2.41), we get


 1 3  1 2  1
r3 − p r3 + q r3 − r = 0
 2  1
r − pr 3 + qr 3 − r = 0
p3 r − q 3 = 0 (2.42)

is the required relation/condition for the roots to be in G.P.


2αγ
(i) Let the roots in H.P. are α, β, γ, where β = α+γ
. We know

α + β + γ = −(−p) = p (2.43)
αβ + βγ + αγ = q (2.44)
αβγ = r (2.45)
2αγ
Also, β=
α+γ
αβ + βγ + αγ = 3αγ. (2.46)

From equation (2.46) and (2.44), we get the value of αγ = 3q . Putting the value of αγ
in equation (2.45), we get the value of β = 3rq . Since β is one of the root of equation
(2.32), therefore it will satisfy the equation. Hence, we get

β 3 − pβ 2 + qβ − r = 0. (2.47)
2.4. APPLICATIONS TO SOLUTION OF EQUATIONS 43

Putting the value of β in equation (2.47), we get


 3  2  
3r 3r 3r
−p +q −r =0
q q q
27r3
   2 
9r p
3
−p + 3r − r = 0
q q
27r2 − 9pq 2 r + 2q 3 = 0 (2.48)
is the required relation/condition for the roots to be in H.P.
Example 2.11. The product of the two roots of the equation
x4 + x3 − 16x2 − 4x + 48 = 0 (2.49)
is 6. Find all the roots of the given equation.
Solution. Let four roots of the given equation (2.49) are α, β, γ, δ. Therefore by using
relation between roots and coefficients we get
α + β + γ + δ = −1 (2.50)
αβ + βγ + αγ + βδ + αδ + γδ = −16
αβ + (α + β)(γ + δ) + γδ = −16 (2.51)
αβγ + αβδ + βγδ + αγδ = 4
αβ(γ + δ) + γδ(α + β) = 4 (2.52)
αβγδ = 48. (2.53)
Also, it is given that αβ = 6. Therefore from equation (2.53), γδ = 8. Putting the value of
αβ and γδ in equation (2.51) and (2.52), we get
6 + (α + β)(γ + δ) + 8 = −16
(α + β)(γ + δ) = −30 (2.54)
6(γ + δ) + 8(α + β) = 4 (2.55)
Let α + β = l and γ + δ = m Then
lm = −30
6m + 8l = 4 (2.56)
Substituting the value of m = − 30l in equation (2.56), we get
 
30
8l + 6 − =4
l
2l2 − l − 45 = 0
2l2 − 10l + 9l − 45 = 0
(2l + 9)(l − 5) = 0
9
l = 5, − (2.57)
2
44LESSON - 2. RELATION BETWEEN THE ROOTS AND COEFFICIENTS OF EQUATIONS

Taking l = 5, we get m = −6. Thus the value of α + β = 5 and γ + δ = −6 (Note:


α + β = − 92 and γ + δ = 60
9
does not satisfy equation (2.50). )
By taking α + β = 5 and αβ = 6, we get

6
α+ =5
α
α2 − 5α + 6 = 0
α = 2, 3 (2.58)

Taking α = 2, we get β = 3 and taking α = 3, we get β = 2. Also by taking γ + δ = −6


and γδ = 8, we get

8
γ+ = −6
γ
γ 2 + 6γ + 8 = 0
γ = −4, −2 (2.59)

Taking γ = −4, we get δ = −2 and taking δ = −2, we get γ = −4. Thus the four roots of
equation (2.49) are given as 2, 3, −4, −2.

In-text Exercise 2.2. Solve the following questions:

(a) Solve the equation 8x3 − 14x2 + 7x − 1 = 0, whose roots are in G.P.

(b) Solve the equation x3 − 5x2 − 16x + 80 = 0, the sum of two of its roots being zero.

(c) Solve the equation x3 − 9x2 + 23x − 15 = 0, whose two roots are in the ratio of 3 : 5.

(d) Solve the equation 2x3 + x2 − 7x − 6 = 0, given that the difference of two of its roots
is 3.

(e) Solve the equation x4 + 2x3 − 21x2 − 22x + 40 = 0, sum of two of its roots being
equal to the sum of the other two.

2.5 Summary
In the end of the chapter, we know

1. The relation between roots and coefficients of an polynomial equation.

2. Using relation between roots and coefficients how to find relation between roots of a
polynomial equation.

3. Using relation between roots, how to find all the roots of the given equation.
2.6. SELF ASSESSMENT EXERCISE 45

2.6 Self Assessment Exercise


1. Solve the equation x3 − 6x2 + 11x − 6 = 0, whose roots are in A.P.
2. Determine the roots of the equation
2x3 − 4x2 − 2x + 4 = 0,
whose two roots are equal in magnitude and opposite in sign.
3. The roots of the equation 3x3 − x2 − 3x + 1 = 0, are in H.P. Find the roots.
4. If the roots of the equation
x3 − px2 + qx − r = 0,
3r
be in harmonic progression, show that the mean root is q
.
5. If the sum of two roots of the equation
4x4 − 24x3 + 31x2 + 6x − 8 = 0,
be zero, find all the roots of the equation.
6. Solve the equation
x4 + 15x3 + 70x2 + 120x + 64 = 0,
whose roots are in geometric progression (G.P.).
7. Solve the equation
x3 − 5x2 − 2x + 24 = 0,
given that the product of two of its roots is 12.
8. Find a necessary condition for the roots of the equation ax3 + bx2 + cx + d = 0 to
be in A.P.
9. The equation
3x4 − 25x3 + 50x2 − 50x + 12 = 0,
has two roots whose product is 2. Find all the roots.
10. Solve the equation
2x4 − 15x3 + 35x2 − 30x + 8 = 0,
given that the product of two roots is equal to the product of the other two roots.
11. Solve the equation
x3 − 9x2 + 14x + 24 = 0,
two of whose roots are in the ratio of 3 : 2.
12. Find the condition for a cubic equation to have a pair of roots.
46LESSON - 2. RELATION BETWEEN THE ROOTS AND COEFFICIENTS OF EQUATIONS

2.7 Solutions to In-text Exercises


Exercise 2.1
(a) Since α + β = 13, and α − β = 17. From both the equation, we can find the value
of α and β. Hence, by using αβ = k, we can obtain the value of k.
(b) Since α + β = − 37 and αβ = − 32 , therefore
α β α2 +β 2
(i) β
+ α
= αβ
= − 61
6
α2 β2 (α+β)3 −3αβ(α+β) 67
(ii) β
+ α
= αβ
= 9
.
Exercise 2.2
(a) Three roots in G.P. are taken as ar , a, ar. Using relation between coefficient and roots
of a cubic equation, we get a = 12 , r = 12 . Hence the three roots are 1, 12 , 41 .
(b) Let the three roots of the given equation are α, β, γ, where α + β = 0. Since
α+β+γ =5
∴ γ = 5.
Also
αβγ = −80
α.(−α).5 = −80
α = ±4
Hence the roots of the equation are 4, −4, 5.
(c) Let the three roots are 3α, 5α, β. Then
3α + 5α + β = 9 (2.60)
15α2 + 8αβ = 23 (2.61)
15α2 β = 15 (2.62)
23
On solving equation (2.60) and (2.61), we get α = 1 or α = 49 . α = 23
49
does not
satisfied the equation. Hence by taking α = 1, we get β = 1, thus the roots are
obtained as 3, 5&1.
(d) Let α, β, γ be the three roots of the given equation. Since α − β = 3, α + β + γ =
− 12 , αβγ = 3. On solving we get α = 2, β = −1, γ = − 23 .
(e) Let the four roots are α, β, γ, δ. Then
α+β =γ+δ (2.63)
α+β+γ+δ = −2 (2.64)
(α + β)(γ + δ) + αβ + γδ = −21 (2.65)
(α + β)γδ + (γ + δ)αβ = 22 (2.66)
αβγδ = 40 (2.67)
2.7. SOLUTIONS TO IN-TEXT EXERCISES 47

On solving equation (2.63) and (2.64) we get α + β = γ + δ = −1 Also using


equation (2.65) αβ + γδ = −22. Solving it with other equation, we get α = 1, β =
−2, γ = 4, δ = −5.

Suggested Readings
1. Burnside, W.S., & Panton, A.W. (1979). The Theory of Equations. Vol. 1. Eleventh
Edition, (Fourth Indian Reprint. S. Chand & Co. New Delhi), Dover Publications,
Inc.

2. Dickson, Leonard Eugene (2009). First Course in the Theory of Equations. John Wi-
ley & Sons, Inc. The Project Gutenberg eBook (http://www.gutenberg.org/ebooks/29785).
Lesson - 3

De Moivre’s Theorem and its


Applications

Structure
3.1 Learning Objectives 48
3.2 Introduction 49
3.3 Complex Numbers 49
3.4 De Moivre’s Theorem 54
3.5 Applications of De Moivre’s Theorem 60
3.5.1 To find roots of complex number 61
3.5.2 Expansions of cos nθ and sin nθ in terms of powers of cos θ and
sin θ 64
3.5.3 Expansions of cosn θ and sinn θ in terms of sines and cosines of
multiples of θ 65
3.6 nth Roots of Unity 66
3.7 Summary 68
3.8 Self-Assessment Exercises 69
3.9 Solutions to In-text Exercises 69

3.1 Learning Objectives


Students will be able to learn

• De Moivre’s theorem for integer and rational index.

• application of fractional index De Moivre’s theorem for finding the roots of a com-
plex number.

• nth root of unity and its properties

48
3.2. INTRODUCTION 49

3.2 Introduction
In order to compute powers of complex numbers, we must consider the process of repeated
multiplication. The process of repeated multiplication give rise a pattern. This pattern
is the core of the theorem named after the French mathematician Abraham De Moivre.
De Moivre’s theorem gives a formula for computing powers of complex numbers. In this
chapter we will explain the main concept of De- Moivre’s theorems for integer and rational
index. Also nth roots of unity and their properties along with some examples are explained
in details.

3.3 Complex Numbers


Complex numbers are the numbers that are expressed in the form of z =√ a + ib where
a, b ∈ R. Here ‘i’ is the imaginary number called ‘iota’ and its value is −1. Complex
numbers has two parts, a which is called the real part and is denoted by Re(z) and another
is b called the imaginary part and is denoted by Im(z). If the Im(z)=0, then we say that the
complex number z is purely real and if the Re(z)=0, then we say that the complex number
is purely imaginary.

Example 3.1. 2 + 3i, 5 + 7i, −9 + 12i, −6 − 8i are the examples of complex numbers

Figure 3.1: Graph of a complex number in argand plane.

This diagram represent how a complex number is represented geometrically in the 2-d
plane.

Conjugate of a complex number


A complex conjugate of a complex number is another complex number that has the same
real part as the original complex number and the imaginary part has the same magnitude
but opposite sign. Conjugate of a complex number z = a + ib is a − ib which is denoted
by z̄.
50 LESSON - 3. DE MOIVRE’S THEOREM AND ITS APPLICATIONS

Power of i

Since i = −1

i2 = −1
i3 = i2 .i = −i
i4 = i2 .i2 = −1. − 1 = 1
1 1 i
i−1 = = × = −i
i i i

Algebra of complex numbers


Let’s understand the different algebra of complex number one by one

Equality of complex number


Let z1 = a1 + ib1 and z2 = a2 + ib2 are two complex numbers. Then z1 = z2 if and only if

Re(z1 ) = Re(z2 )
Im(z1 ) = Im(z2 ).

Addition of complex numbers


Let z1 = a1 + ib1 and z2 = a2 + ib2 are two complex numbers, then sum of two complex
number is calculated as

z1 + z2 = (a1 + ib1 ) + (a2 + ib2 )


= (a1 + a2 ) + i(b1 + b2 )

Difference of complex numbers


Let z1 = a1 + ib1 and z2 = a2 + ib2 are two complex numbers, then difference of two
complex number is calculated as

z1 − z2 = (a1 + ib1 ) − (a2 + ib2 )


= (a1 − a2 ) + i(b1 − b2 )

Multiplication of complex numbers


Let z1 = a1 + ib1 and z2 = a2 + ib2 are two complex numbers, then multiplication of two
complex number is calculated as

z1 × z2 = (a1 + ib1 ) × (a2 + ib2 )


= (a1 a2 − b1 b2 ) + i(a2 b1 + a1 b2 )

Example 3.2. Let z1 = 1 + 7i and z2 = 4 + 5i, then z1 × z2 ?


Solution. z1 × z2 = (1 + 7i) × (4 + 5i) = 4 + 28i + 5i + 35i2 = 4 + 33i − 35 = 33i − 31.
3.3. COMPLEX NUMBERS 51

Division of complex numbers


Let z1 = a1 +ib1 and z2 = a2 +ib2 are two complex numbers, then division of two complex
number is calculated as
z1 (a1 + ib1 )
=
z2 (a2 + ib2 )
z1 (a1 + ib1 ) (a2 − ib2 )
= × (On rationalization)
z2 (a2 + ib2 ) (a2 − ib2 )
z1 (a1 a2 + b1 b2 ) i(a2 b1 − a1 b2 )
= + .
z2 (a22 + b22 ) (a22 + b22 )
3+2i
Example 3.3. Express 4−i
in the form of x + iy.

Solution. Here
3 + 2i (3 + 2i)(4 + i)
=
4−i (4 − i)(4 + i)
12 − 2 + i(8 + 3)
=
16 + 1
10 + 11i
=
17
10 11
= +i .
17 17

Example 3.4. Find the value of (1 + 2i)3

Solution.

(1 + 2i)3 = (1 + (2i)3 + 3.2i + 3.2i.2i)


= (1 − 8i + 6i − 12)
= (−11 − 2i)

Polar form of a complex number


Polar form of a complex number is another way to represent a complex number. In this
form, we find the real and imaginary components in terms of r and θ, where r is the length
of the vector and θ is the angle made with the positive direction of x axis. Let

x + iy = r(cos θ + i sin θ)

Equating the real and imaginary parts, we get

x = r cos θ (3.1)
y = r sin θ (3.2)
52 LESSON - 3. DE MOIVRE’S THEOREM AND ITS APPLICATIONS

Figure 3.2: Graph of polar form of a complex number.

By squaring and adding equation (3.1) and (3.2), we get

x2 + y 2 = r2 sin2 θ + r2 cos2 θ
x2 + y 2 = r 2
p
r = x2 + y 2

Here we take the positive value of r, as r denotes the length of the vector. Also dividing
equation (3.2) by equation (3.1), we get
y r sin θ
= = tan θ (3.3)
x r cos θ 
y
θ = tan−1 (x ̸= 0) (3.4)
x
p
r = x2 + y 2 is called the modulus of the complex number while θ = tan−1 xy is called


the amplitude or arguments of the complex number.


Note: Since there are many values of θ which satisfies the equation θ = tan−1 xy . There-


fore, the value which lies between −π < θ ≤ π is called the principal value of the ampli-
tude. We shall generally take the principal value of θ.

Example 3.5. Express 3 − i in the polar form or standard form.

Solution. Let

3 − i = r(cos θ + i sin θ)

Equating the real and imaginary parts, we get



3 = r cos θ (3.5)
−1 = r sin θ (3.6)
3.3. COMPLEX NUMBERS 53

By squaring and adding equation (3.5) and (3.6), we get



( 3)2 + (−1)2 = r2 sin2 θ + r2 cos2 θ
4 = r2
r=2

Also dividing equation (3.6) by equation (3.5), we get

−1 r sin θ
√ = = tan θ
3 r cos θ
 
−1 −1
θ = tan √ (3.7)
3
π
θ=−
6
Thus, the modulus of the complex number is 2 and argument is − π6 and the polar form is

written as 3 − i = 2 cos − π6 + i sin − π6 .
 

Example 3.6. Express sin ϕ + i cos ϕ in the polar form or standard form.

Solution. Let

sin ϕ + i cos ϕ = r(cos θ + i sin θ)

Equating the real and imaginary parts, we get

sin ϕ = r cos θ (3.8)


cos ϕ = r sin θ (3.9)

By squaring and adding equation (3.8) and (3.9), we get

(sin ϕ)2 + (cos ϕ)2 = r2 sin2 θ + r2 cos2 θ


1 = r2
r=1

Also dividing equation (3.9) by equation (3.8), we get

cos ϕ r sin θ
= = tan θ
sin ϕ r cos θ
θ = tan−1 (cot ϕ) (3.10)
 π 
−1
θ = tan tan −ϕ
2
π
θ = −ϕ
2
π
 is 1 andπargument
Thus, the modulus of the complex number  is 2
− ϕ and the polar form is
π
written as sin ϕ + i cos ϕ = cos 2 − ϕ + i sin 2 − ϕ .
54 LESSON - 3. DE MOIVRE’S THEOREM AND ITS APPLICATIONS

In-text Exercise 3.1. Solve the following


(a) Find the roots of the quadratic equation x2 − 6x + 10 = 0.
(b) Express the following complex number in the polar form

(i) 1 + i 3
(ii) −2 + 2i

(iii) 1 − i 3
(iv) 5 + 2i
2+3i
(c) Express the complex number 1+i
in the form of a + ib. Find its modulus and ampli-
tude.
(d) Show that | cos θ + i sin θ| = 1

3.4 De Moivre’s Theorem


Theorem 3.1 (De Moivre’s theorem for integral index). It states that the integral power of
a complex number in polar form is equal to the product of the same power of modulus with
multiplication of the argument by the same power.
i.e.

(cos θ + ι sin θ)n = (cos nθ + ι sin nθ), where n is any integer

or

(r cos θ + ιr sin θ)n = rn (cos nθ + ι sin nθ)

or

(eιθ )n = einθ

Proof. Case-1 If n is a positive integer.


We will prove the theorem by principle of mathematical induction.

For n = 1 : (r cos θ + ir sin θ)1 = r1 (cos θ + i sin θ) (3.11)

For n = 2 : (r cos θ + ir sin θ)2 =(r cos θ + ir sin θ) ∗ (r cos θ + ir sin θ)


=(r2 cos2 θ + i2 r2 sin2 θ + 2ir2 cos θ sin θ)
=r2 (cos2 θ − sin2 θ + 2i cos θ sin θ) (∵ i2 = −1)
=r2 (cos 2θ + i sin 2θ).

Let us assume that the result is true for n = k, i.e.

For n = k : (r cos θ + ir sin θ)k = rk (cos kθ + i sin kθ) (3.12)


3.4. DE MOIVRE’S THEOREM 55

We will show that the result is true for n = k + 1

(r cos θ + ir sin θ)k+1 =(r cos θ + ir sin θ)k ∗ (r cos θ + ir sin θ)


=rk (cos kθ + i sin kθ)(r cos θ + ir sin θ)(by using equation (3.12))
=rk+1 (cos kθ + i sin kθ)(cos θ + i sin θ)
=rk+1 (cos kθ cos θ − sin kθ sin θ + i cos kθ sin θ + i sin kθ cos θ)
=rk+1 (cos(k + 1)θ + i sin(k + 1)θ)(Using trigonometric identities)

Hence, the result is true for n = k + 1. Thus for all positive integer we have

(r cos θ + ir sin θ)n = rn (cos nθ + i sin nθ).

Case-2 If n is a negative integer.

Since n is a negative integer, therefore n = −k, where k being a positive integer.

(r cos θ + ir sin θ)n =(r cos θ + ir sin θ)−k


1
=
(r cos θ + ir sin θ)k
1
= k (by using equation (3.12))
r (cos kθ + i sin kθ)
cos kθ − i sin kθ
= k (after rationalizing )
r (cos kθ + i sin kθ)(cos kθ − i sin kθ)
cos kθ − i sin kθ
= k
r (cos2 kθ − i2 sin2 kθ)
=r−k (cos kθ − i sin kθ)
=r−k (cos(−k)θ + i sin(−k)θ)
=rn (cos nθ + i sin nθ)

Thus, for all negative integer n we have

(r cos θ + ir sin θ)n = rn (cos nθ + i sin nθ).

Remark. 1. (sin θ + i cos θ) = i(cos θ − i sin θ)

1
2. If z = (cos θ + i sin θ), then z
= (cos θ − i sin θ)
56 LESSON - 3. DE MOIVRE’S THEOREM AND ITS APPLICATIONS

Proof.
1 1
=
z (cos θ + i sin θ)
1 (cos θ − i sin θ)
= ×
(cos θ + i sin θ) (cos θ − i sin θ)
(cos θ − i sin θ)
=
cos2 θ + sin2 θ
=(cos θ − i sin θ)

1
3. = (cos θ + i sin θ) (by rationalization)
(cos θ − i sin θ)
4. (cos θ + i sin θ)−n = cos nθ − i sin nθ

Proof.

(cos θ + i sin θ)−n = cos(−nθ) + i sin(−nθ)


= cos nθ − i sin nθ
∵ cos(−θ) = cos θ
sin(−θ) = − sin θ

5. (cos θ − i sin θ)−n = cos nθ + i sin nθ


6. (cos θ − i sin θ)n = cos nθ − i sin nθ
7. (cos θ1 + i sin θ1 )(cos θ2 + i sin θ2 ) = cos(θ1 + θ2 ) + i sin(θ1 + θ2 )

Proof.

(cos θ1 + i sin θ1 )(cos θ2 + i sin θ2 ) = cos θ1 cos θ2 − sin θ1 sin θ2 + i(sin θ1 cos θ2 + cos θ1 sin θ2 )
= cos(θ1 + θ2 ) + i sin(θ1 + θ2 )

8. (cos θ1 + i sin θ1 )(cos θ2 − i sin θ2 ) = cos(θ1 − θ2 ) + i sin(θ1 − θ2 )


9. (cos θ1 + i sin θ1 )(cos θ2 + i sin θ2 ).....(cos θn + i sin θn ) = cos(θ1 + θ2 ...... + θn ) +
i sin(θ1 + θ2 ...... + θn )
Example 3.7. Find the modulus and the argument of the complex number
(cos 4θ + i sin 4θ)7
z=
(cos θ + i sin θ)5
3.4. DE MOIVRE’S THEOREM 57

Solution. Given
(cos 4θ + i sin 4θ)7
z= (3.13)
(cos θ + i sin θ)5
We know, by De-Moivre’s theorem,

(cos θ + i sin θ)n = (cos nθ + i sin nθ)

Using this in equation (3.13)

cos(4 × 7)θ + i sin(4 × 7)θ


z=
(cos 5θ + i sin 5θ)
cos 28θ + i sin 28θ
=
(cos 5θ + i sin 5θ)
=(cos 28θ + i sin 28θ)(cos 5θ − i sin 5θ) (using remark (iii))
= cos(28 − 5)θ + i sin(28 − 5)θ (using remark (viii))
=(cos 23θ + i sin 23θ).

Thus,

z = (cos 23θ + i sin 23θ) (3.14)

Comparing equation (3.14) by z = x + iy, we get x = cos 23θ , y = sin 23θ. So, modulus
and arguments are:
p
r = x2 + y 2
p
= cos2 23θ + sin2 23θ
=1

y
−1
ϕ = tan
x 
−1 sin 23θ
= tan
cos 23θ
−1
= tan (tan 23θ)
=23θ

Also we know that polar form of complex number z is z = r(cos ϕ + i sin ϕ). Comparing
this with equation (3.14), we get the value of modulus is 1 and value of argument is 23θ.

Example 3.8. Express the following in the form of x + iy and also find the modulus and
argument of the complex number

(1 + cos θ + i sin θ)4


z= .
(sin θ + i cos θ)2
58 LESSON - 3. DE MOIVRE’S THEOREM AND ITS APPLICATIONS

Solution. To apply De Moivre’s theorem, we need to convert it in the form of (cos θ +


i sin θ)n . For this, we know 1 + cos θ = 2 cos2 2θ , and sin θ = 2 sin 2θ cos 2θ

(1 + cos θ + i sin θ)4


z=
(sin θ + i cos θ)2
4
2 cos2 2θ + 2i sin 2θ cos 2θ
=
i2 (cos θ − i sin θ)2
4 4
2 cos 2θ cos 2θ + i sin 2θ

=
−(cos 2θ − i sin 2θ)
4
2 cos 2θ cos( 4θ ) + i sin( 4θ

2 2
)
=−
(cos 2θ − i sin 2θ)
 4
θ
= − 2 cos (cos(2θ) + i sin(2θ)) (cos 2θ + i sin 2θ)
2
 
4 θ
= −16 cos (cos(4θ) + i sin(4θ)) .
2

Thus,
 
θ
4
z= −16 cos (cos(4θ) + i sin(4θ)) . (3.15)
2

We know that polar form of complex number z is z = r(cos θ +  i sin θ). Comparing this
4 θ
with equation (3.15), we get the value of modulus is −16 cos 2 and value of argument is
4θ.

Example 3.9. If yp = cos ( 2πp ) + i sin ( 2πp ). Prove that y1 .y2 .y3 ..........∞ = −1

Solution.
π  π 
y1 = cos + i sin
 2π  2π 
y2 = cos 2
+ i sin 2
 2π   2π 
y3 = cos + i sin
23 23
..
. (3.16)

 π   π   π  π 
∴ y1 .y2 .y3 ..........∞ = cos + i sin . cos 2
+ i sin 2
. (3.17)
2 2  2
π 2
 π 
cos + i sin ...∞
π 23 23
π π   π π π 
= cos + 2 + 3 ... + i sin + 2 + 3 ...
2 2 2 2 2 2
= cos(S) + i sin(S) (3.18)
3.4. DE MOIVRE’S THEOREM 59

π π π π
where S = 2
+ 22
+ ....
23 π
Since this series form a infinite G.P. whose first term is a = 2
π
and common ratio is r = 22
π = 21 . Thus the sum of S = π
2
+ π
22
+ π
23
... = a
1−r
= 2
1− 12
= π.
2

Hence y1 .y2 .y3 ..........∞ = cos(S) + i sin(S)


= cos(π) + i sin(π)
= − 1. (3.19)
1 1
Example 3.10. If z = cos θ + i sin θ, then prove that z p + zp
= 2 cos pθ and z p − zp
=
2i sin pθ.

Solution. Since

z = cos θ + i sin θ
p
z = cos pθ + i sin pθ (3.20)
1
= cos pθ − i sin pθ (3.21)
zp
Thus, adding equation (3.20) and (3.21), we get

1
zp + = 2 cos pθ
zp
By subtracting equation (3.20) and (3.21), we get

1
zp − = 2i sin pθ.
zp
In-text Exercise 3.2. Solve the following

(a) If a = cos α + i sin α and b = cos β + i sin β, prove that


 
a−b α−β
= i tan .
a+b 2


(b) Find the value of ( 3 + i)4 .
n
(c) Prove that (1 + i)n + (1 − i)n = 2 2 +1 cos( nπ
4
).

(d) If two roots of the equation x2 − 2x + 2 = 0 are α and β, prove that


n
 nπ 
αn − β n = 2 2 +1 i sin .
4

Theorem 3.2 (De Moivre’s theorem for rational index). It states that if n is a ratio-
nal/fractional number i.e.(n = pq , where p & q be integers and q ̸= 0 ), then one of
the values of (cos θ + i sin θ)n is (cos nθ + i sin nθ).
60 LESSON - 3. DE MOIVRE’S THEOREM AND ITS APPLICATIONS

Proof. By De Moivre’s theorem for integral index, we know


 q  
θ θ qθ qθ
cos + i sin = cos + i sin .
q q q q
= (cos θ + i sin θ)
 
θ θ
Thus qth power of cos + i sin q q
is (cos θ + i sin θ).
 
i.e. cos θq + i sin θq is one of the qth roots of (cos θ + i sin θ).
  1
θ θ
i.e. cos q + i sin q is one of the value of (cos θ + i sin θ) q .

Raising both side the pth power, we have


 p p
θ θ
cos q + i sin q is one of the value of (cos θ + i sin θ) q .
  p
or cos pq θ + i sin pq θ is one of the value of (cos θ + i sin θ) q .
Hence one of the values of (cos θ + i sin θ)n is (cos nθ + i sin nθ), where n is rational
number.

Remark. (i) In case of integer power of a complex number, there is one and only one
value of (cos θ + i sin θ)n = (cos nθ + i sin nθ). But in case of rational power (n =
p
q
), there are q values of (cos θ + i sin θ)n one of which is (cos pq θ + i sin pq θ) or
(cos nθ + i sin nθ).

(i) This theorem is also true for irrational numbers. In that case there are infinite number
of values.
p
Theorem 3.3. Show that all the values of (cos θ + i sin θ) q are given by (cos(2rπ + θ) pq +
i sin(2rπ + θ) pq ), where r = 0, 1, 2...q − 1.

or
1
Show that (cos pθ + i sin pθ) q has total q different values.

Proof. Proof is omitted here.

3.5 Applications of De Moivre’s Theorem


The primary use of De Moivre’s Theorem is to obtain the roots of a complex number, so-
lution of some equations and relationship between some the powers and angle of trigono-
metric expressions etc.
3.5. APPLICATIONS OF DE MOIVRE’S THEOREM 61

3.5.1 To find roots of complex number


1
Example 3.11. Write down all the values of (1 + i) 4 .

Solution. Let the complex number

(1 + i) = r(cos θ + i sin θ)

then on comparing real and imaginary parts, we have

r cos θ = 1 (3.22)
r sin θ = 1 (3.23)

Squaring on both sides of the equations (3.22) and (3.23) and adding, we get

r2 cos2 θ + r2 sin2 θ = 1 + 1
r2 = 2

r= 2

Dividing equation (3.23) by equation (3.22), we get

tan θ = 1
π
θ = tan−1 (1) =
4

Thus, (1 + i) = 2(cos π4 + i sin π4 ). Now using De Moivre’s theorem for rational index

1
√  π π  14
(1 + i) 4 = 2 cos + i sin
 4 4
√ 1

π 1  π 1
= ( 2) cos 2kπ +
4 . + i sin 2kπ + . , where k = 0, 1, 2, 3.
4 4 4 4

1
Thus, the four values of (1 + i) 4 are
√ 1h π  π i
for k=0, ( 2) 4 cos + i sin
 16  16 
√ 1 9π 9π
for k=1, ( 2) 4 cos + i sin
16 16
√ 1
    
17π 17π
for k=2, ( 2) 4 cos + i sin
16 16
√ 1
    
25π 25π
for k=3, ( 2) 4 cos + i sin
16 16
62 LESSON - 3. DE MOIVRE’S THEOREM AND ITS APPLICATIONS
√ 1
Note:- It was observed that the 3rd value ( 2) 4 cos 17π + i sin 17π
 
16 16
can be written as
√ 1 √ 1h 
    
17π 17π π  π i
( 2) 4 cos + i sin = ( 2) 4 cos π + + i sin π
16 16 16 16
Since cos(π + θ) = − cos θ
sin(π + θ) = − sin θ
√ 1 √ 1h
    
17π 17π π  π i
∴ ( 2) cos
4 + i sin = ( 2) − cos
4 − i sin (3.24)
16 16 16 16
√ 1h π  π i
= −( 2) 4 cos + i sin
16 16
which is negative of the first value.
√ 1
Similarly ( 2) 4 cos 25π 25π
 
16
+ i sin 16
can be written as
√ 1 √ 1
         
25π 25π 9π 9π
( 2) cos
4 + i sin = −( 2) cos
4 + i sin
16 16 16 16
1
Thus the four values of (1 + i) 4 can be written as
√ 1h π  π i
±( 2) 4 cos + i sin
  16  16
√ 1 9π 9π
±( 2) 4 cos + i sin
16 16
√ 1
or simply, ±( 2) 4 cos pπ pπ
 
16
+ i sin 16
, where p = 1 and 9.
√ 3
3 4
Example 3.12. Find all the values of ( 21 + i 2
) .
Solution. Let the complex number
√ ! 34
1 3
+i = r(cos θ + i sin θ).
2 2

On comparing real and imaginary parts, we have


1
r cos θ = (3.25)
2

3
r sin θ = (3.26)
2
Squaring on both sides of the equations (3.25) and (3.26) and adding, we get
 2 √ !2
1 3
r2 cos2 θ + r2 sin2 θ = +
2 2
r2 = 1
r = +1
3.5. APPLICATIONS OF DE MOIVRE’S THEOREM 63

(as r is the length of the vector so it can not be negative). Also dividing equation (3.26) by
equation (3.25), we get

r sin θ 3/2
=
r cos θ 1/2

tan θ = 3
√ π
θ = tan−1 ( 3) = .
3
Therefore, the complex number
√ !
1 3  π π
+i = 1 cos + i sin .
2 2 3 3

Now,
√ ! 34 
1 3 π π  34
+i = cos + i sin
2 2 3 3
 1
π π 3 4
= cos + i sin
3 3
 π   π  41
= cos 3 + i sin 3 (using De Moivre’s theorem for integer index)
3 3
1
= (cos π + i sin π) 4 .

Using De Moivre’s theorem for rational index, we get


 
1 1 1
(cos π + i sin π) 4 = cos (2kπ + π) . + i sin (2kπ + π) . where k = 0, 1, 2, 3.
4 4
Thus the obtained four values are
h π   π i
for k=0, P1 = cos + i sin
 4  4 
3π 3π
for k=1, P2 = cos + i sin
4 4
    
5π 5π
for k=2, P3 = cos + i sin
4 4
    
7π 7π
for k=3, P4 = cos + i sin
4 4
The product of above four values is
   
π 3π 5π 7π π 3π 5π 7π
P1 .P2 .P3 .P4 = cos + + + + i sin + + +
4 4 4 4 4 4 4 4
= cos(4π) + i sin(4π)
=1
64 LESSON - 3. DE MOIVRE’S THEOREM AND ITS APPLICATIONS

3.5.2 Expansions of cos nθ and sin nθ in terms of powers of cos θ and


sin θ
By De Moivre’s theorem, we have
(cos θ + i sin θ)n = cos nθ + i sin nθ (3.27)
Also,
n
(cos θ + i sin θ)n = C 0 cosn θ + nC 1 cosn−1 θ(i sin θ)1 + nC 2 cosn−2 θ(i sin θ)2 +
... + nC n cosn−n θ(i sin θ)n

(cos θ + i sin θ)n = cosn θ + i nC 1 cosn−1 θ sin θ − nC 2 cosn−2 θ sin2 θ + ... + (i)n sin nθ
(3.28)
From equation (3.27) and (3.28), we get
cos nθ + i sin nθ = cosn θ + i nC 1 cosn−1 θ sin θ − nC 2 cosn−2 θ sin2 θ + ... + (i)n sin nθ
(3.29)
On comparing real and imaginary part, we get
cos nθ = cosn θ − nC 2 cosn−2 θ sin2 θ + ... (3.30)
sin nθ = nC 1 cosn−1 θ sin θ − nC 3 cosn−3 θ sin3 θ + ... (3.31)
This is the required relation between cos nθ and sin nθ and powers of cosθ and sin θ.
Example 3.13. Write down the expansion of cos 5θ, sin 5θ and tan 5θ in terms of powers
of cosθ, sin θ. and tan θ.
Solution. Putting n = 5 in equation (3.29), we get
cos 5θ + i sin 5θ = cos5 θ + i 5C 1 cos4 θ sin θ − 5C 2 cos3 θ sin2 θ − i 5C 3 cos2 θ sin3 θ
+ 5C 4 cos1 θ sin4 θ + i 5C 5 sin5 θ (3.32)
On comparing real and imaginary part, we get
cos 5θ = cos5 θ − 5C 2 cos3 θ sin2 θ + + 5C 4 cos1 θ sin4 θ
sin 5θ = 5C 1 cos4 θ sin θ − 5C 3 cos2 θ sin3 θ + 5C 5 sin5 θ
On solving, we get
cos 5θ = cos5 θ − 10 cos3 θ sin2 θ + 5 cos θ sin4 θ (3.33)
sin 5θ = 5 cos4 θ sin θ − 10 cos2 θ sin3 θ + sin5 θ (3.34)
Dividing equation (3.34) by equation (3.33), we get
sin 5θ 5 cos4 θ sin θ − 10 cos2 θ sin3 θ + sin5 θ
tan 5θ = = (3.35)
cos 5θ cos5 θ − 10 cos3 θ sin2 θ + 5 cos θ sin4 θ
Dividing the numerator and denominator of equation (3.35) by cos5 θ, we get
5 tan θ − 10 tan3 θ + tan5 θ
tan 5θ = (3.36)
1 − 10 tan2 θ + 5 tan4 θ
3.5. APPLICATIONS OF DE MOIVRE’S THEOREM 65

3.5.3 Expansions of cosn θ and sinn θ in terms of sines and cosines of


multiples of θ
Let
z = cos θ + i sin θ (3.37)
then
1
= cos θ − i sin θ (3.38)
z
Also
z n = cos nθ + i sin nθ (3.39)
then
1
= cos nθ − i sin nθ. (3.40)
zn
Adding and subtracting equation (3.39) and (3.40), we get
1
z n + n = 2 cos nθ (3.41)
z
1
z n − n = 2i sin nθ (3.42)
z
Also from equation (3.37) and (3.38), we have
 n
1
z+ = (2 cos θ)n (3.43)
z
 n
1
z− = (2i sin θ)n (3.44)
z
Expanding the left hand side of equation (3.43) by binomial expansion and using the equa-
tion (3.41), we get the required relation between cosn θ and cosines of multiples of θ. Sim-
ilarly expanding the left hand side of equation (3.44) and using the equation (3.42), we get
the required relation between sinn θ and sines of multiples of θ
Example 3.14. Prove that cos 6θ + 6 cos 4θ + 15 cos 2θ + 10 = 32 cos6 θ.
Solution. Expanding the left hand side of equation (3.43) by binomial expansion for (n =
6)
 6
1
z+ = (2 cos θ)6
z
   2  3  4
6 6 5 1 6 4 1 6 3 1 6 2 1
z + C 1z + C 2z + C 3z + C 4z
z z z z
 5  6
1 1
+ 6C 5 z + 6C 6 = 26 cos6 θ
z z
 2  4  6
6 4 2 1 1 1
z + 6z + 15z + 20 + 15 +6 + = 26 cos6 θ
z z z
     
1 1 1
z + 6 + 6 z + 4 + 15 z + 2 + 20 = 26 cos6 θ
6 4 2
z z z
66 LESSON - 3. DE MOIVRE’S THEOREM AND ITS APPLICATIONS

Using equation (3.41), we get

2 cos 6θ + 12 cos 4θ + 30 cos 2θ + 20 = 26 cos6 θ


cos 6θ + 6 cos 4θ + 15 cos 2θ + 10 = 32 cos6 θ

is the required relation between cosn θ and cosines of multiples of θ.

In-text Exercise 3.3. Solve the following


1
(a) Find all the values of (i) 8 .
√ 2
(b) Find all the values of ( 3 − i) 5 .

(c) Expand cos 7θ, sin 7θ in terms of powers of cosθ and sin θ.

(d) Prove that −32 sin6 θ = cos 6θ − 6 cos 4θ + 15 cos 2θ − 10.

3.6 nth Roots of Unity


The solution of the equation z n = 1 , for positive values of integer , called the nth roots of
unity or a root of unity also known as De Moivre’s number is a complex number that gives
the value 1 when raised to some positive integer power n. Mathematically, the solution of
1
z n = 1 or z = (1) n is called the nth roots of unity.

Example 3.15. Find the nth roots of unity.

Solution. Given z n = 1
1
z =(1) n
1
=(cos 0 + i sin 0) n
   
1 1
= cos (2rπ + 0). + i sin (2rπ + 0). , ( By using theorem 3.3)
n n
where r = 0, 1, 2, ......n − 1
   
2rπ 2rπ
= cos + i sin , where r = 0, 1, 2, ......n − 1.
n n

Thus, nth roots of unity are    


2(n−1)π 2(n−1)π
cos 0+i sin 0, cos 2π 2π 4π 4π
   
n
+i sin n
, cos n
+i sin n
, ..., cos n
+i sin n

Let α = cos 2π + i sin 2π 4π 4π


   
n n
then α2 = cos n
+ i sin n
. Therefore, nth roots of unity
are found as 1, α, α2 ..., αn−1 .

Example 3.16. Show that the nth roots of unity form a series in G.P.
3.6. nth ROOTS OF UNITY 67

Solution. Since nth roots of unity are 1, α, α2 ...αn−1 , where α = cos 2π + i sin 2π
 
n n
.
The given series form a G.P. whose first term is 1 and common ratio is α. The sum of the
series is found as
1.(1 − αn )
1 + α + α2 ...αn−1 =
1 − α n 
1 − cos 2π + i sin 2π

n n
=
1 − cos 2π + i sin 2π
  
n n
1 − cos 2nπ + i sin 2nπ
  
n n
=
1 − cos 2π + i sin 2π
  
n n
(1 − 1)
=
1 − cos 2π + i sin 2π
  
n n
= 0.

    n
n 2π 2π
α = cos + i sin
n n
    
2nπ 2nπ
= cos + i sin
n n
= cos(2π) + i sin(2π)
=1

Sum of the nth roots of unity can also be found by using relation between roots and coeffi-
cients as 1, α, α2 ...αn−1 are the roots of z n − 1 = 0. Thus

coefficient ofz n−1


Sum of the roots = 1 + α + α2 ... + αn−1 = − = 0.
coefficient ofz n
constant term
product of the roots = 1.α.α2 .α3 + ... + αn−1 = (−1)n = (−1)n−1 .
coefficient ofz n
Thus the sum of all the n roots of unity is 0 and product of all the roots of unity is (−1)n−1 ,
also αn = 1.

Example 3.17. Solve the equation z 4 + 1 = 0

Solution. Given z 4 + 1 = 0
1
z =(−1) 4
1
=(cos π + i sin π) 4
   
1 1
= cos (2rπ + π). + i sin (2rπ + π). , ( By using theorem 3.3)
4 4
where r = 0, 1, 2, 3.
   
(2r + 1)π (2r + 1)π
= cos + i sin , where r = 0, 1, 2, 3.
4 4
68 LESSON - 3. DE MOIVRE’S THEOREM AND ITS APPLICATIONS

Thus the four roots of z 4 + 1 = 0, are


π  π  1 1
when r=0, cos + i sin = √ + i√
4 4 2 2
   
3π 3π 1 1
when r=1, cos + i sin = −√ + i√
4 4 2 2
   
5π 5π 1 1
when r=2, cos + i sin = −√ − i√
4 4 2 2
   
7π 7π 1 1
when r=3, cos + i sin = √ − i√
4 4 2 2

i.e. ± √12 ± i √12 are the four roots of the given equation.

Remark. 1. The nth roots of unity lie on the circumference of the circle, whose radius
is equal to 1 and center is the origin (0, 0).

2. The sum of all the roots of unity is 0 and product of all the roots of unity is (−1)n−1 ,
also αn = 1.

3. The nth roots of unity 1, α, α2 ...αn−1 are in geometric progression called G.P.

In-text Exercise 3.4. Solve the following

(a) Find the cube roots of unity. Also find the sum and product of the roots.

(b) Solve the equation z 6 + z 3 + 1 = 0.

(c) Solve the equation x7 − x4 + x3 − 1 = 0.

(d) Find the fifth roots of unity and verify that the obtained roots form a geometric pro-
gression.

3.7 Summary
In the end of the chapter, we know

1. De Moivre’s theorem for integer and fraction index.

2. De Moivre’s theorem gives a formula for computing powers of complex numbers.

3. This theorem is also helpful for discovering correlation between the functions of
numerous angles that are calculated using trigonometry.

4. Using this theorem we can find the nth roots of unity.


3.8. SELF-ASSESSMENT EXERCISES 69

3.8 Self-Assessment Exercises


1. Write the trigonometric representation of 1 + cos α + i sin α.
n
2. If (1 + x)n = C 0 + nC 1 x + nC 2 x2 + ... + nC n xn , where n is a positive integer. Then
prove that
n n
(a) C 0 − nC 2 + nC 4 − nC 6 + ...xn = 2( 2 ) cos( nπ
4
)
n n
(b) C 1 − nC 3 + nC 5 − nC 7 + ...xn = 2( 2 ) sin( nπ
4
)

3. If sin α + sin β + sin γ = 0 and cos α + cos β + cos γ = 0. Prove that

(a) cos 3α + cos 3β + cos 3γ = 3 cos(α + β + γ)


(b) sin 3α + sin 3β + sin 3γ = 3 sin(α + β + γ)
(c) cos(β + γ) + cos(γ + α) + cos(α + β) = 0
(d) sin(β + γ) + sin(γ + α) + sin(α + β) = 0
(cos θ+i sin θ)7
4. Express (cos 2θ+i sin 2θ)2
in the form of a + ib.
√ 3
5. Write down all the values of (1 + i 3) 5 .

6. Expand sin7 θ cos2 θ in a series of sines of multiple of θ.

7. Prove that 256 sin5 θ cos4 θ = sin 9θ − sin 7θ − 4 sin 5θ + 4 sin 3θ + 6 sin θ.

8. Express cos4 θ in terms of cosines of multiples of θ.

9. Solve the expression z 6 − z 5 + z 4 − z 3 + z 2 − z + 1 = 0.

10. Solve the equation z 9 + z 5 + z 4 + 1 = 0.

11. Find the fifth roots of (−32).

3.9 Solutions to In-text Exercises


Exercise 3.1

(a) Roots obtained by using quadratic formula are x = 3 ± i.


√ √
(b) Complexnumber 5 + 2i has modulus r = 52 + 22 = 29 and argument as θ =
tan−1 52 . Thus the polar form of the given complex number is


      
−1 2 −1 2
5 + 2i = 29 cos tan + i sin tan
5 5
5 + 2i ≈ 5.3(cos(0.38) + i sin(0.38)).
70 LESSON - 3. DE MOIVRE’S THEOREM AND ITS APPLICATIONS

(c)
2 + 3i 1 − i 5+i
z= × = .
1+i 1−i 2

(d) Let z = cos θ + i sin θ, then |z| = cos θ2 + sin θ2 = 1.

Exercise 3.2

(a) Since

a + b = (cos α + cos β) + i(sin α + sin β)


     
α−β α+β α+β
a + b = 2 cos cos + i sin
2 2 2
a − b = (cos α − cos β) + i(sin α − sin β)
   
α−β α+β α+β
a − b = 2i sin cos( ) + i sin
2 2 2
a−b
= i tan α−β

Hence a+b 2
.

(b) Since
√ 4
 h π   π i4
( 3 + i) = 2 cos + i sin
 6  6 

2π 2π
= 16 cos + i sin
3 3

= −8 + i8 3

(c) Since
√  π   π 
1+i= 2 cos + i sin
4 4 
√  π  π
1 − i = 2 cos − i sin
4 4
Therefore
n
  nπ   nπ 
(1 + i)n = 2 2 cos + i sin (3.45)
4 
  nπ 4 
 nπ
n
n
(1 − i) = 2 2 cos − i sin (3.46)
4 4
After adding both the equation (3.45) and (3.46), we get the required result.

(d) In the above question by taking α = 1 + i and β = 1 − i and subtracting equation


(3.46) from (3.45), we get the required result.

Exercise 3.3
3.9. SOLUTIONS TO IN-TEXT EXERCISES 71

1 1
(a) (i) 8 = (cos π2 + i sin π2 ) 8 . Now using De Moivre’s theorem for rational index
  
1 π 1  π 1
(i) = cos 2kπ +
8 . + i sin 2kπ + . where k = 0, 1, 2, 3, 4, 5, 6, 7.
2 8 2 8
 π  π
= cos (4k + 1) + i sin (4k + 1) where k = 0, 1, 2, 3, 4, 5, 6, 7.
16 16

√ 2 2 2
(b) ( 3 − i) 5 = (2) 5 (cos π6 − i sin π6 ) 5 . Now using De Moivre’s theorem for rational
index

√ 2 2
 π π  15
( 3 − i) = (2) cos − i sin
5 5

 3 3 
2 π 1  π 1
= (2) 5 cos 2kπ − . + i sin 2kπ − . where k = 0, 1, 2, 3, 4.
3 5 3 5

(c) Putting n = 7 in equation (3.29), we get

cos 7θ + i sin 7θ = cos7 θ + i 7C 1 cos6 θ sin θ − 7C 2 cos5 θ sin2 θ − i 7C 3 cos4 θ sin3 θ


+ 7C 4 cos3 θ sin4 θ + i 7C 5 cos2 θ sin5 θ − 7C 6 cos1 θ sin6 θ − i 7C 7 sin7 θ.
(3.47)

On comparing real and imaginary part, we get the required result.

(d) Expanding the left hand side of equation (3.44) by binomial expansion for (n = 6)
 6
1
z− = (2i sin θ)6
z
   2  3  4
6 6 5 1 6 4 1 6 3 1 6 2 1
z − C 1z + C 2z − C 3z + C 4z
z z z z
 5  6
1 1
− 6C 5 z + 6C 6 = −26 sin6 θ
z z
 2  4  6
6 4 2 1 1 1
z − 6z + 15z − 20 + 15 −6 + = −26 sin6 θ
z z z
     
1 1 1
z 6 + 6 − 6 z 4 + 4 + 15 z 2 + 2 − 20 = −26 sin6 θ
z z z

Using equation (3.41), we get

cos 6θ − 6 cos 4θ + 15 cos 2θ − 10 = −32 sin6 θ.

Exercise 3.4
72 LESSON - 3. DE MOIVRE’S THEOREM AND ITS APPLICATIONS

1
(a) To find the cube root of unity we have to solve z 3 − 1 = 0 or z = (1) 3 .
1
z =(1) 3
1
=(cos 0 + i sin 0) 3
   
1 1
= cos (2rπ + 0). + i sin (2rπ + 0). , ( By using theorem 3.3)
3 3
where r = 0, 1, 2
   
2rπ 2rπ
= cos + i sin , where r = 0, 1, 2.
3 3
√ √
3 3
Thus the three roots of unity are 1, − 12 + i 2
, − 21 −i 2
.
(b) Since the given equation is quadratic in z 3 , therefore we get

3 1 3
z =− ±i
2 √2
1 3
z3 = − ± i = r(cos θ ± i sin θ)
2 2
 1
2π 2π 3
z = cos(2rπ + ) ± i sin(2rπ + ) where r = 0, 1, 2.
3 3

(c) Since the given equation is x7 − x4 + x3 − 1 = 0.


x7 − x4 + x3 − 1 = 0
(x4 + 1)(x3 − 1) = 0
(x4 + 1) = 0, (x3 − 1) = 0.
By example 3.17 and cube root of unity, the seven roots are given as

1 3 1 1
1, − ± i , ±√ ± i√ .
2 2 2 2

(d) Fifth roots of unity can be obtained same as the cube roots of unity.

Suggested Readings
1. Burnside, W.S., & Panton, A.W. (1979). The Theory of Equations. Vol. 1. Eleventh
Edition, (Fourth Indian Reprint. S. Chand & Co. New Delhi), Dover Publications,
Inc.
2. Dickson, Leonard Eugene (2009). First Course in the Theory of Equations. John Wi-
ley & Sons, Inc. The Project Gutenberg eBook (http://www.gutenberg.org/ebooks/29785).
3. Prasad, Chandrika (2017). Text Book of Algebra and Theory of Equations. Poth-
ishala Pvt Ltd.
Lesson - 4

Cubic and Biquadratic Equations

Structure
4.1 Learning Objectives 73
4.2 Introduction 74
4.3 Algebraic solution of the cubic equation 74
4.3.1 Cardan’s Method 74
4.3.2 Descartes’ Solution of the Quartic Equation 77
4.4 Transformation of Equations 79
4.4.1 To form an equation whose roots are the negatives of the roots of
a given equation 79
4.4.2 To form an equation whose roots are c times the roots of a given
equation 80
4.4.3 To form an equation whose roots are the reciprocals of the roots
of a given equation 82
4.4.4 To form an equation whose roots are less by or greater by k then
the roots of a given equation 85
4.4.5 To form an equation in which some specific term is missing 86
4.5 Summary 88
4.6 Self Assessment Exercise 88
4.7 Solutions to In-text Exercises 89

4.1 Learning Objectives


After studying this chapter, student should be able to

• better understanding of transformation of equations.

• understand the relationship between roots and the coefficients of the equations.

73
74 LESSON - 4. CUBIC AND BIQUADRATIC EQUATIONS

• apply Cardano’s method for calculating the roots of cubic equations.


• apply Descartes’ method for calculating the roots of biquadratic equations.

4.2 Introduction
Babylonians and other various mathematicians by 2000 B.C.E. devised methods for finding
solutions of cubic equations. During the early and middle 16th centuries mathematicians
discovered formulas for the roots of cubic and biquadratic (fourth degree) polynomials in
terms of the coefficients of the polynomial.
Scipione dal Ferro (1465-1526) is credited with solving cubic equations. The problem
was to find the roots by adding, subtracting, multiplying, dividing and taking roots of ex-
pressions in the coefficients. The original formula was discovered in one basic case but not
published by Ferro. It was rediscovered independently and extended to other cases by Ni-
colo Fontana, who is better known as Tartaglia and G. Cardano’s without permission from
Tartaglia by G. Cardano Ain his book, ”Ars Magna” which was devoted to algebra which
is given as:

4.3 Algebraic solution of the cubic equation


4.3.1 Cardan’s Method
An equation of the form
px3 + qx2 + rx + s = 0
is called the cubic equation i.e., an equation of 3rd degree with real coefficients.
Example 4.1. 1. x3 + x2 + 1 = 0 is a cubic equation with real coefficients.
2. x3 = 1 is an equation of 3rd degree.
3. 6x5 + x2 = 9 is not a cubic equation.
Let the general cubic equation be given by

px3 + qx2 + rx + s = 0 (4.1)

Eliminating the square term with the help of the substitution y = x + q/3p, we obtain
q 3 q q r q s
(y − ) + (y − )2 + (y − ) + = 0
3p p 3p p 3p p
2 3 2
q q 2q q3 qr s
⇒ y 3 − qy 2 + y − + qy 2
− y + + ry − + =0
3p 27p2 3p 9p2 3p p
q2 2q 3 qr
⇒ py 3 + (r − 2 )y + (s + − )=0
3p 27p3 3p
3pr − q 2 27p2 s + 2q 3 − 9pqr
⇒ y3 + y + = 0. (4.2)
3p2 27p3
4.3. ALGEBRAIC SOLUTION OF THE CUBIC EQUATION 75

For example, consider the equation 2x3 − 30x2 + 15x + 30 = 0. Then by substituting
y = x − (−30)/(3 × 2) = x + 5, we obtain
2(y 3 + 15y 2 + 75y + 125) − 30(y 2 + 10y + 25) + 15(y + 5) + 30 = 0
2y 3 + (150 − 300 + 15)y + (250 − 750 + 75 + 30) = 0
2y 3 − 135y − 395 = 0.
Consider the equation (1.2). It can be written as
z 3 + 3Az − 2B = 0
where
q2
A = (r − )
3p
2q 3 qr
B = (s + 2
− ).
27p 3p
Now, let z = u + v and A = −uv, we obtain
(u + v)3 + 3(−uv)(u + v) − 2B = 0
⇒ (u3 + 3u2 v + 3uv 2 + v 3 ) − 3u(u + v) − 3v(u + v) − 2B = 0
⇒ u3 + v 3 − 2B = 0
A3
⇒ u3 − 3 − 2B = 0
u
⇒ u − A3 − 2u3 = 0
6

⇒ u6 − 2Bu3 − A3 = 0.
The above equation
√ is quadratic in u3 , which has the solutions given by
u3 = B ± B 3 + A2 . √
3
Since uv = −A, we obtain that v 3 = − Au3 and u3 = B + B 3 + A2 . This gives that

v 3 = B − B 3 + A2 .
Therefore, we obtain

u3 = B + B 3 + A2 ,

v 3 = B − B 3 + A2

⇒ u = (B + B 3 + A2 )( 1/3) = α,

v = (B − B 3 + A2 )( 1/3) = β.
Three cube roots of u3 and v 3 are given by
ωα, ω 2 α, ωβ, ωβ 2 .
Therefore, the three roots of the given equation (4.3.1) are
q
x1 = α + β − ,
3p

α+β q i 3
x2 = − − + (α − β),
2 3p 2

α+β q i 3
x3 = − − − (α − β).
2 3p 2
76 LESSON - 4. CUBIC AND BIQUADRATIC EQUATIONS

This solution is called Cardan’s solution of the cubic.


Example 4.2. Solve 2x3 + 3x2 + 6x + 4 = 0.
Solution. We shall remove the second term in the given equation by completing the cube
in the following way:
3
x3 + x2 + 3x + 2 = 0
2
1 3 3 1
[(x + ) − x − ] + 3x + 2 = 0
2 4 8
1 3 3 1
⇒ (x + ) − x + 3x + 2 − = 0
2 4 8
1 3 9 15
⇒ (x + ) + x + 3x + = 0.
2 4 8
1
Put x + 2
= y in the above equation, we get
9 1 1 15
y 3 + (y − ) + 3(y − ) = 0
4 2 2 8
9 9 3 15
⇒ y 3 + y − + 3y − + =0
4 8 2 8
9 3 6
⇒ y 3 + y + 3y − + = 0
4 2 8
21 3
⇒ y 3 + y − = 0.
4 4
Let y = m + n where m, n ∈ C. Then
21 3
(m + n)3 +(m + n) − =0
4 4
21 3
m3 + 3mn(m + n) + n3 + (m + n) − =0
4 4
21 3
⇒ m3 + n3 + 3mn(m + n) + (m + n) − =0
4 4
7 3
⇒ m3 + n3 + 3(m + n)(mn + ) − =0
4 4
This gives that mn + 47 = 0. This implies mn = − 47 . This further gives that m3 + n3 = 43
and m3 n3 = 27
64
.
We see that m3 and n3 are the roots of t2 + 74 t + 27
64
= 0. Hence, by quadratic formula,
we find that
q
−7 + 4 49 16
− 27
16
3
m = =α
q8
−7 − 4 49 16
− 27
16
3
n = = β.
8
Hence, the roots of the equation are α + β, αω + βω 2 , αω 2 + βω.
4.3. ALGEBRAIC SOLUTION OF THE CUBIC EQUATION 77

In-text Exercise 4.1. Find the roots of the following cubic equations:
1. x3 − 6x − 9 = 0.
2. x3 − 9x2 + 14x + 24 = 0.

4.3.2 Descartes’ Solution of the Quartic Equation


The equation of general quartic is given by
x4 + ax3 + bx2 + cx + d = 0. (4.3)
Replacing x = z − a/4 in (4.3), we obtain
z 4 + pz 2 + qz + r = 0. (4.4)
(4.3) is called the reduced quartic equation. We claim that (4.4) can be expressed as the
product of two quadratic factors
(z 2 + kz + l)(z 2 − kz + m) = z 4 + (l + m − k 2 )z 2 + k(l − m)z + lm
On equating the coefficients, we obtain
l + m − k2 = p
k(l − m) = q
lm = r.
This gives that
l + m = p + k2,
q
l−m= .
k
If k ̸= 0, from first two equations, we obtain
q
2l = p + k 2 −
k
2 q
2m = p + k +
k
Putting the above values in l · m = r, we have
k 6 + 2pk 4 + (p2 − 4r)k 2 − q 2 = 0. (4.5)
On putting k 2 = t, we get
t3 + 2pt2 + (p2 − 4r)k − q 2 = 0.
This is a cubic equation with at least one positive real root. Thus (4.4) is equivalent to
solving
(z 2 + kz + l)(z 2 − kz + m) = 0
⇒ (z 2 + kz + l) = 0 or (z 2 − kz + m) = 0.
The above quadratic equations will give 4 roots of (4.3).
78 LESSON - 4. CUBIC AND BIQUADRATIC EQUATIONS

Example 4.3. Solve x4 + 8x3 + 9x2 − 8x − 10 = 0.

Solution. The above equation can be rewritten as

3
x4 + 4(2)x3 + 6 x2 + 4(−2)x − 10 = 0.
2

We shall remove the x3 term in the given equation using x = z + h where h = − ab


= − 21 = −2. So we get x = z − 2. Putting this value of x in the given equation, we get

(z − 2)4 + 8(z − 2)3 + 9(z − 2)2 − 8(z − 2) − 10 = 0


⇒ z 4 − 8z 3 + 24z 2 − 32z + 16
+ 8(z 3 − 48z 2 + 96z − 64) + 9(z 2 − 4z + 4) − 8(z − 2) − 10 = 0
⇒ z 4 − 8z 3 + 24z 2 − 32z + 16
+ 8z 3 − 48z 2 + 96z − 64 + 9z 2 − 36z + 36 − 8z + 16 − 10 = 0
⇒ z 4 − 15z 2 + 20z − 6 = 0

Then we write

z 4 − 15z 2 + 20z − 6 = (z 2 + kz + l)(z 2 − kz + m).

This gives that

l + m − k 2 = −15
k(l − m) = 20
lm = −6.

On eliminating l and m from the above equations, we get

(l + m)2 − (l − m)2 = 4lm


20
(k − 15)2 − ( )2 = 4 · −6
k
k 6 − 30k 4 + 249k 2 − 400 = 0.

k 2 = 16 is the root of the above equation in k 2 . Thus, k = 4. Using this value of k, we


obtain l = 3 and m = −2. Therefore, the roots of the given equation are the roots √ of the
equations (x2 + 4x + 3)(x2 − 4x − 2) = 0. This gives √4 roots as 1, 3 and −2 ± i 6. Thus
the roots of the equation are z − 2 i.e. are −1, 1, 2 ± 6.

Example 4.4. Solve x4 − 2x2 + 8x − 3 = 0.

Solution. Since there is no x3 term in the given equation, we write the above equation as

x4 − 2x2 + 8x − 3 = (z 2 + kz + l)(z 2 − kz + m).


4.4. TRANSFORMATION OF EQUATIONS 79

This gives that

l + m − k 2 = −2
k(l − m) = 8
lm = −3.

On eliminating l and m from the above equations, we get

(l + m)2 − (l − m)2 = 4lm


8
(k + 2)2 − ( )2 = 4 · 6
k
k − 4k + 16k 2 − 64 = 0.
6 4

k 2 = 4 is the root of the above equation in k 2 . Thus, k = 2. Using this value of k, we


obtain l = 3 and m = −1. Therefore, the roots of the given equation √ are the roots of√the
equations x2 + 2x − 3)(x2 − 2x − 1) = 0. This gives 4 roots as −1 ± 2 and −1 ± i 2.

In-text Exercise 4.2. Find the roots of the following quartic equations:

1. x4 − 2x2 + 8x − 3 = 0.

2. x4 + 8x3 + 9x2 − 8x − 10 = 0.

4.4 Transformation of Equations


Sometimes without knowing the roots of an equation in terms of its coefficients, we can
transform one symmetric equation into another in which roots of the new equation has
some relation with the roots of the previous equation.

4.4.1 To form an equation whose roots are the negatives of the roots of
a given equation
To form an equation whose roots are the negatives of the roots of a given equation of degree
n, multiply the coefficients of xn , xn−1 , · · · by 1, −1, 1, −1, · · · .
Let p(x) = a0 x + a1 x + a2 x2 + · · · + an xn = 0 Suppose α1 , α2 , · · · , αn are the roots
of p(x) then

p(x) = (x − α1 )(x − α2 ) · · · (x − αn ). (4.6)

Put x = −y, we have

a0 − a1 y + a2 y 2 + · · · + (−1)nan y n = (y + α1 )(y + α2 ) · · · (y + αn ).

Thus the roots are −α1 , +α2 , · · · , −αn . Therefore, the required equation is

a0 − a1 x + a2 x2 + · · · + (−1)n an x = 0
80 LESSON - 4. CUBIC AND BIQUADRATIC EQUATIONS

Note. To form an equation whose roots are the negative of the roots of the given equation,
change the signs of every alternate term of the given equation written in decreasing powers
of x.
Example 4.5. Find the equation whose roots are the negatives of the roots of the equation
x3 − 3x2 + 5x − 4 = 0.
Solution. Putting x = −y in the given equation, we get

(−y)3 − 3(−y)2 + 5(−y) − 4 = 0


⇒ y 3 + 3y 2 + 5y + 4 = 0.

Hence, x3 + 3x2 + 5x + 4 = 0 is the required equation.


Example 4.6. Find the equation whose roots are the roots of the equation x4 − 5x3 + 7x2 −
17x + 11 = 0 with their signs changed.
Solution. Putting x = −y in the given equation, we get

(−y)4 − 5(−y)3 + 7(−y)2 − 17(−y) + 11 = 0


⇒ y 4 + 5y 3 + 7y 2 + 17y + 11 = 0.

Hence, x4 + 5x3 + 7x2 + 17x + 11 = 0 is the required equation.


In-text Exercise 4.3. 1. Find the equation whose roots are the roots of
8x5 − 5x4 + 3x3 + x2 + x + 8 = 0 with their sign changed.
2. Find the equation whose roots are the negative of the roots of
x4 + 21x3 − x2 + 10x + 11 = 0.

4.4.2 To form an equation whose roots are c times the roots of a given
equation
Let p(x) = a0 x + a1 x + a2 x2 + · · · + an xn = 0 Suppose α1 , α2 , · · · , αn are the roots of
p(x) then

p(x) = (x − α1 )(x − α2 ) · · · (x − αn ) (4.7)

Put x = cy in (1.1), we have


Therefore the required equation is

y  y  y  y 
p = a0 − α1 − α2 · · · − αn
c c c c
Thus the roots of p( yc ) are cα1 , cα2 , · · · , cαn . Therefore, the required equation is
y  y n  y n−1
p( ) = a0 + a1 + · · · + an = 0
c c c
a0 y n + ca1 y n−1 + c2 a2 y n−2 + · · · + cn an = 0.
4.4. TRANSFORMATION OF EQUATIONS 81

Note. The equation whose roots are c times the roots of a given equation, multiply the
coefficients and the constant term by 1, c, c2 , · · · , cn−1 and cn to the given equation arranged
in the increasing powers of x.

Example 4.7. Find the equation whose roots are three times those of the equation x3 +
2x2 − 3x + 1 = 0.
y
Solution. Let y be the root of the required equation then y = 3x. Putting x = 3
in the
given equation, we get
 y 3  y 2 y
+2 − +1=0
3 3  3 2  3
y y y
+2 − + 1 = 0.
27 9 3
3 2
y + 6y − 9y + 27 = 0.

Hence, y 3 + 6y 2 − 9y + 27 = 0 is the required equation.

Example 4.8. Find the equation whose roots are five times those of the equation 4x4 +
6x3 + 7x2 − x + 2 = 0.
y
Solution. Let y be the root of the required equation then y = 5x. Putting x = 5
in the
given equation, we get
 y 4  y 3  y 2 y
4 +6 +7 − +2=0
5 4  5 5   5
y3 y2

y y
4 +6 +7 − + 2 = 0.
625 125 25 5
4 3 2
4y + 30y + 175y − 125y + 625 = 0.

Hence, 4y 4 + 30y 3 + 175y 2 − 125y + 625 = 0 is the required equation.

Example 4.9. Remove the fractional coefficient from the equation x3 − 4x2 + 41 x − 19 = 0.
y
Solution. Taking y = cx or x = c
in the given equation, we get
 y 3  y 2
1y 1
−4 − =0
+
c c 4c 9
1 1
⇒ y 3 − 4y 2 c + yc2 − = 0
4 9
1 1
⇒ y 3 − 4y 2 c + 2 c2 − 2 c3 = 0.
2 3
The value of c for which the fraction will disappear is c = 2 · 3 = 6. Putting this value of
c = 6, we get

y 3 − 24y 2 + 9y − 24 = 0

Hence, the required equation is x3 − 24x2 + 9x − 24 = 0.


82 LESSON - 4. CUBIC AND BIQUADRATIC EQUATIONS

Example 4.10. Transform the equation 72x3 − 54x2 + 45x − 7 = 0 into equation with
integral coefficients and unity of the leading coefficient.
Solution. The given equation can be written as
54 2 45 7
x3 − x + x− = 0.
72 72 72
y
Taking y = cx or x = c
in the given equation, we get
 y 3 54  y 2 45 y 7
− + − =0
c 72 c 72 c 72
3 5 7
⇒ y 3 − y 2 c + yc2 − c3 = 0.
4 8 72
The above equation is with the leading coefficient as unity. If we take c = 22 · 3, then we
get

y 3 − 9y 2 + 90y − 168 = 0

Hence, the required equation is x3 − 9x2 + 90x − 168 = 0.


In-text Exercise 4.4. 1. Find the equation whose roots are three times the roots of
x3 + 11x2 + 13x + 2 = 0.

2. Transform the equation 12x3 − 48x2 + 56x − 8 = 0 into equation with integral
coefficients and unity of the leading coefficient.

4.4.3 To form an equation whose roots are the reciprocals of the roots
of a given equation
Reciprocal Equations are those equations which remains unchanged when x is replaced by
its reciprocal.
Let p(x) = a0 x + a1 x + a2 x2 + · · · + an xn = 0. Suppose α1 , α2 , · · · , αn are the roots
of p(x) then

p(x) = (x − α1 )(x − α2 ) · · · (x − αn ) (4.8)

Put y = x1 i.e., x = y1 in (4.8), we have


Therefore the required equation is

      
1 1 1 1
p = − α1 − α2 · · · − αn
y y y y
 n  n−1   
1 1 an 1 1 1
a0 + a1 + · · · + an = − α1 − α2 · · · ( − αn )
y y yn y y y
    
n an−1 n−1 an−2 n−2 a1 a0 1 1 1
⇒y + y + y + ··· + y + = − α1 − α2 · · · − αn .
an an an an y y y
4.4. TRANSFORMATION OF EQUATIONS 83

Thus the condition obtained are


an−1
= a1 ,
an
an−2
= a2 ,
an
··· ,
1
= an . (4.9)
an
The last equation (??) gives a2n = 1. This implies that an = +1 or an = −1.
Reciprocal equations are classified into two classes according to an = 1 oran = −1.
Case I. First class of reciprocal equations are the equations in which the coefficients of
the corresponding terms taken from the beginning and end are equal in magnitude and have
the same signs i.e.,
a1 = an−1 , a2 = an−2 , · · · , an−1 = a1 .
Case II. Second class of reciprocal equations, in which corresponding terms counting
from the beginning and end are equal in magnitude but different in signs i.e.,
a1 = −an−1 , a2 = −an−2 , · · · , an−1 = −a1 .
Note. 1. When the degree of the equation is even then one of the condition becomes
am = −am or am = 0. So in reciprocal equations of the second class whose degree
is even the middle term is absent and roots occur in pairs i.e., α1 , α11 etc.
2. When the degree of the equation is odd then there must be a root which is its own
reciprocal.
3. Since the reciprocal equation is of first class or second class and −1 or 1 is the root.
So the equation can be divided by x + 1 or x − 1 and outcome is the reciprocal
equation of even degree and of the first class.
4. In equations of the second class of even degree x2 − 1 is a factor, since the equation
may be written in the form
xn − 1 + a1 x(xn−2 − 1) + · · · = 0.
By dividing by x2 − 1, this is also reducible to a reciprocal equation of the first class
of even degree. Hence all reciprocal equations may be reduced to those of the first
class whose degree is even
Example 4.11. Solve x4 − 10x3 + 16x2 − 10x + 1 = 0.
Solution. The given equation is a standard reciprocal equation. Dividing throughout by x2 ,
we get
10 1
x2 − 10x + 16 − + 2 =0
x x
1 1
⇒ (x2 + 2 ) − 10(x + ) + 16 = 0.
x x
84 LESSON - 4. CUBIC AND BIQUADRATIC EQUATIONS

Putting y = x + x1 , we get

y 2 − 2 − 10y + 16 = 0
⇒ y 2 − 10y + 14 = 0
⇒ (y − 5)(y − 2) = 0
⇒ y = 5 or 2.

Case I:

y=5
1
⇒x+ =5
x
2
⇒ x − 5x + 1 = 0

5 ± 25 − 4
⇒x=
2√
5 ± 21
⇒x= .
2
Case II.

y=2
1
⇒x+ =2
x
2
⇒ x − 2x + 1 = 0
⇒ (x − 1)2 = 0
⇒ x = 1, 1.
√ √
Hence, the roots are 1, 1, 5+2 21 , 5−2 21 .
Example 4.12. Solve 5x3 − 21x2 − 21x + 5 = 0.
Solution. This is an odd degree reciprocal equation of first class. Therefore, −1 is the root
of the equation and (x + 1) is the factor of the given equation. Dividing the given equation
by (x + 1), we obtain 5x2 − 26x + 5 as the quotient. Solving the quotient, we obtain the
roots as 5, 51 . Thus −1, 51 , 5 are the roots of the given equation.
Example 4.13. Find the equation whose roots are the reciprocal of the roots of the equation
x4 − 5x3 + 3x2 − 2x + 1 = 0.
Solution. Let y be the root of the required equation then y = x1 . Putting x = 1
y
in the given
equation, we get
 4  3  2  
1 1 1 1
−5 +3 −2 +1=0
y y y y
⇒ 1 − 5y + 3y 2 − 2y 3 + y 4 = 0.

Hence, y 4 − 2y 3 + 3y 2 − 5y + 1 = 0 is the required equation.


4.4. TRANSFORMATION OF EQUATIONS 85

Example 4.14. Find the equation whose roots are the reciprocal of the roots of the equation
5x5 − 5x3 − 2x + 4 = 0.

Solution. Let y be the root of the required equation then y = x1 . Putting x = 1


y
in the given
equation, we get
 5  3  
1 1 1
5 −5 −2 +4=0
y y y
⇒ 4y 5 − 2y 4 − 5y 2 + 5 = 0.

Hence, 4x4 − 2x3 − 5x + 5 = 0 is the required equation.

Example 4.15. If α1 , α2 , α3 are the roots of x3 + ax2 + bx + c = 0, find the equation whose
roots are α12 , α22 , α32 .

Solution. Our aim is to find the equation whose roots are square of the roots of

x3 + ax2 + bx + c = 0.

Taking y = x2 in the above equation, we get

xy − ay + bx − c = 0
⇒ (y + b)x = ay + c
⇒ (y + b)2 x2 = (ay + c)2
⇒ (y + b)2 y = (ay + c)2
⇒ (y + b)2 y − (ay + c)2 = 0.

4.4.4 To form an equation whose roots are less by or greater by k then


the roots of a given equation
To form an equation whose roots are less by or greater by k then the roots of a given
equation. ( i.e., Diminishing or increasing the roots by k ). Let p(x) = a0 x + a1 x + a2 x2 +
· · · + an xn = 0 Suppose α1 , α2 , · · · , αn are the roots of p(x) then

p(x) = (x − α1 )(x − α2 ) · · · (x − αn ) (4.10)

Put y = x − k i.e., x = y + k in (1.7), we have


Therefore the required equation is

p(y + k) = a0 (y + k − α1 )(y + k − α2 ) · · · (y + k − αn )

Thus the roots of p(y + k) are α1 − k, α2 − k, · · · , αn − k. Therefore, the required equation


is

p(y + k) = a0 (y + k)n + a1 (y + k)n−1 + · · · + an = 0.


86 LESSON - 4. CUBIC AND BIQUADRATIC EQUATIONS

Expanding using binomial theorem and combining like terms, we get an equation of the
form

b0 y n + b1 y n−1 + · · · + bn = 0. (4.11)

Replacing y = x − k, we obtain

b0 (x − k)n + b1 (x − k)n−1 + · · · + bn = 0. (4.12)

Now, equation p(x) and (4.12) represents the same equation. Dividing equation (4.12)
continuously by (x − k), we obtain the remainders as b0 , b1 , · · · , bn .
Substituting these in (4.11), we get the required equation.

Example 4.16. Find the equation whose roots are the roots of the equation 4x3 − 2x2 +
7x − 3 = 0 each decreased by 2.

Solution. Let y = x − 2 so that x = y + 2. Putting the value of x in the given equation, we


get

4x3 − 2x2 + 7x − 3 = 0
⇒ 4(y + 2)3 − 2(y + 2)2 + 7(y + 2) − 3 = 0
⇒ 4y 3 + 24y 2 + 48y + 32 − 2y 2 − 8 − 8y + 7y + 14 − 3 = 0
⇒ 4y 3 + 22y 2 + 47y + 35 = 0.

Therefore, 4x3 + 22x2 + 47x + 35 = 0 is the required equation.

In-text Exercise 4.5. 1. Find the equation whose roots are the roots of x4 +10x2 +3x+
12 = 0 each decreased by 3.

2. Find the equation whose roots are the roots of x3 + x2 + 4x − 5 = 0 each decreased
by 2.

4.4.5 To form an equation in which some specific term is missing


Let

p(x) = a0 x + a1 x + a2 x2 + · · · + an xn = 0. (4.13)

Suppose it is required to remove the second term of the given equation. Diminish the
roots of the equation by k. Put y = x − k or x = y + k in the above equation, we get

a0 (y + k)n + a1 (y + k)n−1 + · · · + an = 0
⇒ a0 y n + (na0 k + a1 )y n−1 + · · · + an = 0.

Now, to remove the second term from the given equation, we must have na0 k + a1 = 0.
a1
This gives k = − na0
.
4.4. TRANSFORMATION OF EQUATIONS 87

Example 4.17. Solve the equation x4 − 12x3 + 49x2 − 78x + 40 = 0 by removing its
second term.

a1 −12
Solution. We compute that h = a0
= 4·1
= −3. Replacing x by (x + 3) in the given
equation, we obtain

(x + 3)4 − 12(x + 3)3 + 49(x + 3)2 − 78(x + 3) + 40 = 0


⇒ x4 + 4(3x3 ) + 6(9x2 ) + 4(27x) + 81
− 12(x3 + 3(3x2 ) + 3(9x) + 27) + 49(x2 + 9 + 6x) − 78x − 234 + 40 = 0
⇒ x4 + 12x3 + 54x2 + 108x + 81
− 12x3 − 108x2 − 324x − 324 + 49x2 + 441 + 294x − 78x − 234 + 40 = 0
⇒ x4 − 5x2 + (562 − 558) = 0
⇒ x4 − 5x2 + 4 = 0
⇒ (x2 − 4)(x2 − 1) = 0
⇒ x2 = 4, 1
⇒ x = ±2, ±1.

Hence (x + 3) gives 1, 5, 4, 2 as the roots of the given equation.

Example 4.18. Find the equation whose roots are the roots of x4 −5x3 +3x2 −12x+11 = 0
each diminished by 4.

x − 2| 1 −5 3 −14 +15
4 −4 12 −20
1 −1 −1 −2 −5
4 12 60
Solution. 1 3 11 58
4 28
1 7 39
4
1 11
Hence, the required equation is given by x4 + 11x3 + 39x2 + 58x − 5 = 0.

Example 4.19. Find the equation whose roots are the roots of x5 − 3x4 − x3 + 4x2 − 13x +
15 = 0 each diminished by 2.
88 LESSON - 4. CUBIC AND BIQUADRATIC EQUATIONS

x − 2| 1 −3 −1 4 −13 15
2 −2 −6 −4 −34
1 −1 −3 −2 −17 −19
2 2 −2 −8
1 1 −1 −4 −25
2 6 −10
Solution. 1 3 −5 −14
2 10 10
1 5 5 −4
2 14
1 7 19
2
1 9
Hence, the required equation is given by x5 + 9x4 + 19x3 − 4x2 − 25x + 19 = 0.

4.5 Summary
1. Cardan’s method is used to find the roots of the cubic equation.

2. Descartes’ method is used to find the roots of the biquadratic equation.

3. Transformation of equation is transforming one symmetric equation into another in


which roots of the new equation has some relation with the roots of the previous
equation.

4.6 Self Assessment Exercise


1. Solve the following cubic equations using Cardan’s method:

(a) x3 − 2x2 + 5x + 6 = 0.
(b) 2x3 + x2 − 8x − 4 = 0.
(c) x3 − 3x + 12 = 0.

2. Solve the following biquadratic equations using Descartes’ method:

(a) x4 + 4x3 + 2x2 + 6x + 8 = 0.


(b) x4 − 3x3 + 2x2 − 11 = 0.
(c) x4 + 7x2 + 5x + 3 = 0.

3. Form an equation whose roots are four times the roots of the equation x3 − 6x2 +
8x − 3 = 0.

4. Form an equation whose roots are five times the roots of the equation x3 + 2x2 + 3x +
1 = 0.
4.7. SOLUTIONS TO IN-TEXT EXERCISES 89

5. Form an equation whose roots are the negatives of the roots of the equation 3x3 −
2x2 + x − 4 = 0.

6. Form an equation whose roots are the reciprocal of the roots of the equation x4 −
15y 3 − 2y + 11 = 0.

7. Write an equation whose roots are diminished by 3 of the roots of the equation x4 −
5x3 + 7x2 − 5x + 1 = 0.

8. Solve the equation x3 + 5x2 − 4x + 7 = 0 by removing its second term.

9. Solve the equation x4 − 6x3 + 11x2 − 6x + 4 = 0 by removing its second term.

10. Remove the fractional coefficient from the equation


2 1
x3 − x2 + x − 1 = 0.
3 2

4.7 Solutions to In-text Exercises


Excercise 1.1
√ √
1. 3, 12 (−3 + 3i), 21 (−3 − 3i.

2. −1, 4, 6.

Excercise 1.2
√ √
1. Roots are −1 ± 2, −1 ± i 2.

2. Roots are 1, 3, −2 ± 6.

Exercise 1.3

1. −8x5 + 5x4 + 3x3 − x2 + x − 8 = 0.

2. x4 − 21x3 − x2 − 10x + 11 = 0.
Lesson - 5

Symmetric Functions

Structure
5.1 Learning Objectives 90
5.2 Introduction 91
5.3 Symmetric Functions 91
5.4 Fundamental Theorem on Symmetric Functions 94
5.5 Rational Functions Symmetric in all but One of the Roots. 95
5.6 Sums of Like Powers of the Roots 96
5.7 Newton’s Theorem on the Sums of the powers of the roots 97
5.8 Theorems relating to Symmetric Functions 98
5.9 Computation of Symmetric Functions 100
5.10 Summary 100
5.11 Self Assessment Excercise 100

5.1 Learning Objectives


After studying this chapter, student should be able to

• define symmetric functions and can check whether the given function is symmetric
or not.

• implement the fundamental theorem on symmetric functions.

• compute symmetric functions.

• better understanding of transformation of equations with the help of symmetric func-


tions.

90
5.2. INTRODUCTION 91

5.2 Introduction
In algebra, theory of equations is the study of algebraic equations given by a polynomial.
Around 18th century, there were two questions of major interest first was the roots of
the equation and second was the relationship between the roots and the coefficients of the
equation. Theory of equations are also studied through the special type of functions called
symmetric functions. The theory of Symmetric functions play an important role in the
representation theory of Groups and Combinatorics. Many mathematicians uses symmetry
function theory to study the permutations and cycle structure. It has many applications in
mathematics and mathematical physics like lie algebra, random matrix theory and symme-
tries.

5.3 Symmetric Functions


Definition 5.1. Symmetric functions of the roots of an equation are those functions in
which all the roots are alike involved, so that the expression remains same in the value
when any two of the roots are interchanged.
This means that if an expression has two roots say α1 and α2 then replacing α1 by α2
in the expression or vice-versa, the value of the expression is unchanged or unaltered.
Symmetric functions are denoted by the letter Σ attached to one term of it with the help
of which entire expression can P be written down. For example, if α1 and α2 are the roots of
the quadratic equation then α1 = α1 + α2 is the symmetric function.
Also, if α1 , α2 , α3 are the roots of the cubic equation then the expression Σα1 represent
the symmetric function α1 +α2 +α3 and Σα1 3 α2 3 means the sum α1 3 α2 3 +α2 3 α3 3 +α3 3 α1 3 .
P P
Note. α1 , α1 α2 , α1 α2 α3 are called the elementary symmetric functions. P PIn gen-
eral,
P the elementary symmetric functions of α 1 , α2 , · · · , αn are given by α1 , α1 α2 ,
α1 α2 α3 , · · · , α1 α2 · · · αn .
Note. α12 α2 + α22 α3 + α12 α3 is not a symmetric function as interchanging of α1 and α2 will
change the given function.
Example 5.1. If α1 , α2 , α3 are the roots of the equation x3 − ax2 + bx − c = 0, find the
values of
P 2
1. α1 α2 α3
P 2 2
2. α1 α2

3. (α1 + α2 )(α2 + α3 )(α3 + α1 )


Solution.

α1 + α2 + α3 = a,
α1 α2 + α2 α3 + α3 α1 = b,
α1 α2 α3 = c.
92 LESSON - 5. SYMMETRIC FUNCTIONS

(i).
X
α12 α2 α3 = α12 α2 α3 + α22 α1 α3 + α32 α1 α2
X
⇒ α12 α2 α3 = α1 α2 α3 (α1 + α2 + α3 )
X
⇒ α12 α2 α3 = ca.

(ii).

(α1 α2 + α2 α3 + α3 α1 )2 = α12 α22 + α22 α32 + α32 α12 + 2(α12 α2 α3 + α22 α1 α3 + α32 α1 α2 )
X
⇒ b2 = α12 α22 + 2α1 α2 α3 (α1 + α2 + α3 )
X
⇒ α12 α22 + 2 = 2ca − b2 .

(ii). (α1 + α2 )(α2 + α3 )(α3 + α1 ) = (a − α3 )(a − α1 )(a − α2 ).


Since α1 , α2 , α3 are the roots of the given equation, so

x3 − ax2 + bx − c = (x − α1 )(x − α2 )(x − α3 ). (5.1)

Putting x = a in (2.1), we get

(a − α1 )(a − α2 )(a − α3 ) = a3 − a3 + ab − c
⇒ (α1 + α2 )(α2 + α3 )(α3 + α1 ) = ab − c.

Example 5.2. If α1 , α2 , α3 are the roots of x3 + ax2 + bx + c = 0, find the values of


P 1
(a) α1
.
P 2
(b) α1 .
P 2
(c) α1 α2 .
P 3
(d) α1 .
Solution. We have

α1 + α2 + α3 = −a,
α1 α2 + α2 α3 + α3 α1 = b,
α1 α2 α3 = −c.

= 1 + 1 + 1 = α1 α2 +α 2 α3 +α3 α1
P 1
(a). Consider = − cb .
P α12 α12 α22 α32 α1 α2 α3
(b). Consider α1 = α1 + α2 + α3 . Now,

(α1 + α2 + α3 )2 = α12 + α22 + α32 + 2α1 α2 + 2α2 α3 + 2α3 α1


= α12 + α22 + α32 + 2(α1 α2 + α2 α3 + α3 α1 )
⇒ (−a)2 = α12 + α22 + α32 + 2b
⇒ α12 + α22 + α32 = a2 − 2b.
5.3. SYMMETRIC FUNCTIONS 93

(c).
X
α12 α2 = α12 α2 + α22 α3 + α32 α1 + α22 α1 + α12 α3 + α32 α1
X X
⇒ α1 α1 α2 = (α1 + α2 + α3 )(α1 α2 + α2 α3 + α3 α1 )
X
= α12 α2 + 3α1 α2 α3
X X X
⇒ α12 α2 = α1 α1 α2 − 3α1 α2 α3
= −ab + 3c.
(d). Consider
X X
α1 α12 = (α1 + α2 + α3 )(α12 + α22 + α32 )
= −a(a2 − 2b) − (−ab + 3c)
= −a3 − 3ab − 3c
X X X
⇒ α13 = α1 α12 − α12 α2
= −a(a2 − 2b) − (−ab + 3c)
= −a3 − 3ab − 3c.
Example 5.3. If α1 , α2 , α3 , α3 are the roots of x4 + ax3 + bx2 + cx + d = 0, find the values
of
P 1
(a) α1
.
P 2
(b) α1 .
P 2
(c) α1 α2 .
P 2
(d) α1 α2 α4 .
P 2 2
(e) α1 α2 .
P 4
(f) α1 .
Solution.
α1 + α2 + α3 + α4 = −a,
α1 α2 + α2 α3 + α3 α1 = b,
α1 α2 α3 + α2 α3 α4 + α3 α4 α1 = −c
α1 α2 α3 α4 = d.
(i).
X 1 1 1 1 1
= + + +
α1 α1 α2 α3 α4
α1 α2 α3 + α2 α3 α4 + α3 α4 α1
=
α1 α2 α3 α4
−c
= .
d
94 LESSON - 5. SYMMETRIC FUNCTIONS

(ii).
X
α12 = α12 + α22 + α32 + α42
= (α1 + α2 + α3 + α4 )2 − 2(α1 α2 + α2 α3 + α3 α4 + α4 α1 + α4 α2 + α1 α4 )
= a2 − bc

(iii).
X X X X
α1 α2 = α12 α2 + α1 α2 α3
X X X X
⇒ α12 α2 = α1 α2 − α1 α2 α3
X
⇒ α12 α2 = −ab + c.

(iv).
X X X
α1 α1 α3 α4 = α12 α2 α3 + 4α1 α2 α3 α4
X X X
⇒ α12 α2 α3 = α1 α1 α3 α4 − 4α1 α2 α3 α4
= (−a)(−c) − 4d
= ac − 4d.

(v).
X X X
( α1 α2 )2 = α12 α22 + 2 α12 α2 α3 + 6α1 α2 α3 α4
X X X
⇒ α12 α22 = ( α1 α2 )2 − 2 α12 α2 α3 − 6α1 α2 α3 α4
X
⇒ α12 α22 = b2 − 2(ac − 4d) − 6d
= b2 − 2ac + 2d.

(vi).
X X X X
α12 α12 = α14 + 2 α12 α22
X X X X
⇒ α14 = α12 α12 − 2 α12 α22
X
⇒ α14 = (a2 − 2b)2 − 2(b2 − 2ac + 2d)
= a4 + 2b3

5.4 Fundamental Theorem on Symmetric Functions


Any polynomial symmetric in x1 , · · · , xn is equal to an integral rational function, with
integral coefficients, of the elementary symmetric functions
X X X
E1 = x1 , E2 = x1 x2 , E3 = x1 x2 x3 , · · · , En = x1 x2 · · · xn
5.5. RATIONAL FUNCTIONS SYMMETRIC IN ALL BUT ONE OF THE ROOTS. 95

and the coefficients of the given polynomial. In particular, any symmetric polynomial with
integral coefficients is equal to a polynomial in the elementary symmetric functions with
integral coefficients.
The equivalent form of the fundamental theorem is given by
Any polynomial symmetric in the roots of an equation, xn − E1 xn−1 + E2 xn−2 + · · · +
(−1)n En = 0,
is equal to an integral rational function, with integral coefficients, of the coefficients of
the equation and the coefficients of the polynomial.
For n = 2,
E1 (x1 , x2 ) = x1 + x2 , E2 (x1 , x2 ) = x1 x2 .
For n = 3,
x31 + x32 = E(x1 , x2 )3 − 3E1 (x1 , x2 )E2 (x1 , x2 ).

Example 5.4. Let us write the symmetric function α12 + α22 + α32 in terms of elementary
symmetric functions.

E1 = α1 + α2 + α3
E2 = α1 α2 + α2 α3 + α3 α1
E3 = α1 α2 α3 .

Now,

α12 + α22 + α32 = (α1 + α2 + α3 )2 − 2(α1 α2 + α2 α3 + α3 α1 )


⇒ α12 + α22 + α32 = E12 − 2E2 .

5.5 Rational Functions Symmetric in all but One of the


Roots.

If P is a rational function of the roots of an equation f (x) = 0 of degree n and if P


is symmetric in n − 1 of the roots, then P is equal to a rational function, with integral
coefficients, of the remaining root and the coefficients of f (x) and P .

P α21 +α22
Example 5.5. If α1 , α2 , α3 are the roots of x3 +ax2 +bx+c = 0, find the value of α1 +α2
.
96 LESSON - 5. SYMMETRIC FUNCTIONS
P
Solution. α1 + α2 + α3 = −a, α1 α2 = b, α1 α2 α3 = −c.
X α2 + α2 (α1 + α2 )2 − 2α1 α2 2α1 α2
1 2
= = (α1 + α2 ) −
α1 + α2 α1 + α2 α1 + α2
X α2 + α2 X 
α1 α2 α2 α3 α1 α3

1 2
= (α1 + α2 ) − 2 + +
α1 + α2 α1 + α2 α2 + α3 α1 + α3
P
X 2 α1 α2 (α1 + α3 )(α2 + α3 )
=2 α1 −
(α1 + α2 )(α1 + α3 )(α2 + α3 )
2 α1 α2 ( α1 α2 + α12 )
P P
= −2a −
(−a − α1 )(−a − α2 )(−a − α3 )
2 α1 α2 (b + α12 )
P
= −2a +
(−a − α1 )(−a − α2 )(−a − α3 )
P
2(bb + α1 α2 α3 α1 )
= −2a + 3 P P
(a + a2 α1 + a α1 α2 + α1 α2 α3 )
2(b2 + ac)
= −2a + 3
(a − a3 + ab − c)
2b2 + 4ac − 2a2 b
= .
(ab − c)

Example 5.6. If α1 , α2 , α3 are the roots of the equation x3 + ax2 + bx + c = 0 then find
the equation whose roots are α1 − α21α3 , α2 − α31α1 , α3 − α11α2 .

Solution. Let
1 α1 1
y = α1 − = α1 − = α1 (1 + )
α2 α3 α1 α2 α3 c
1
⇒ y = x(1 + )
c
cy
⇒x= .
1+c
Putting this value of x in the given equation, we get
 3  2
cy cy cy
+a +b +c=0
1+c 1+c 1+c
c2 y 3 + ac(1 + c)y 2 + b(1 + c)2 y + (1 + c)3 = 0.

5.6 Sums of Like Powers of the Roots


If α1 , α2 , · · · , αn are (x) = xn + a1 xn−1 + a2 xn−2 + · · · + an = 0
Pthe2 equation fP
Pthe roots of
We write s1 = α1 , s2 = α1 and sk = α1k . We obtain

sk + a1 sk−1 + a2 sk−2 + · · · + an sk−n = 0, k > n. (5.2)

Equations given by (2.2) is called Newton’s Identities


5.7. NEWTON’S THEOREM ON THE SUMS OF THE POWERS OF THE ROOTS 97

5.7 Newton’s Theorem on the Sums of the powers of the


roots
The sums of the similar powers of the roots of an equation can be expressed rationally in
terms of the coefficients. In other words,
If α1 , α2 , α3 , · · · , α1 are the roots of the equation xn +A1 xn−1 +A2 xn−2 +· · ·+An = 0,
and Sk = α1k k + α2k + · · · + αnk . Then Sk + Sk−1 A1 + · · · + Ak−1 S1 + kAk = 0 if r ≤ n
and Sk + Sk−1 A1 + · · · + Ak−1 S1 + Sk−n An = 0 if k > n.

Example 5.7. If α1 is an imaginary root of the equation x7 − 1 = 0 then form the equation
whose roots are α1 + α16 , α12 + α15 , α13 + α14 .

Solution. Let a = α1 +α16 , b = α12 +α15 , c = α13 +α14 be the roots of the new equation. Then
the new equation is (x−a)(x−b)(x−c) = 0 i.e., x3 −(a+b+c)x2 +(ab+bc+ca)x−abc = 0
Now,

a + b + c = α1 + α16 + α12 + α15 + α13 + α14


= α1 (1 + α1 + α12 + α13 + α14 + α15 )
α1 (α16 − 1)
=
α1 − 1
(α7 − α1 )
= 1
α1 − 1
(1 − α1 )
= = −1
α1 − 1
and

ab + bc + ca = (α1 + α16 )(α12 + α15 ) + (α12 + α15 )(α13 + α14 ) + (α13 + α14 )(α1 + α16 )
= α13 + α16 + α18 + α111 + α15 + α16 + α18 + α19 + α14 + α19 + α15 + α110 .

Since α1 is a root of x7 –1 = 0, we have α17 = 1,

ab + bc + ca = α13 + α16 + α1 + α14 + α15 + α16 + α1 + α12 + α14 + α1 + α15 + α13


= 2(α1 + α12 + α13 + α14 + α15 + α16 )
= −2(1) = −2.

Next,

abc = (α1 + α16 )(α12 + α15 )(α13 + α14 )


= (α13 + α16 + α18 + α11 1)(α13 + α14 )
= (α16 + α17 + α19 + α110 + α111 + α112 + α114 + α115 )
= α16 + α12 + α13 + α14 + α15 + 1 + α1 ) = −1.

Thus the required equation is


x3 + x2 –2x + 1 = 0.
98 LESSON - 5. SYMMETRIC FUNCTIONS

Example 5.8. Find the value in terms of the coefficients of sum of the reciprocals of the
roots of the equation xn + a1 xn−1 + a2 xn−2 + · · · + an−1 x + an = 0.

Solution. We know that

α1 α2 · · · αn−1 + · · · + α2 α3 · · · αn = (−1)n−1 an−1 (5.3)


α1 α2 α3 · · · αn = (−1)n an . (5.4)

Dividing (2.4) by (2.3), we get

1 1 1 an−1
+ + ··· + =−
α1 α2 αn an
X 1 an−1
=− .
α1 an

5.8 Theorems relating to Symmetric Functions


1. The sum of the exponents of all the roots in any term of any symmetric function of the
roots is equal to the sum of the suffixes in each term of the corresponding value in terms of
the coefficients.
The suffix of each coefficient in those equations is equal to the degree in the roots of the
corresponding function of the roots ; hence in any product of any powers of the coefficients
the sum of the suffixes must be equal to the degree in all the roots of the corresponding
function of the roots.
2. When an equation is written with binomial coefficients, the expression in terms of the
coefficients for any symmetric function of the roots, which is a function of their differences
only, is such that the algebraic sum of the numerical factors of all the terms in it is equal to
zero.

Example 5.9. Find the value of the symmetric function (α1 −α2 )2 +(α2 −α3 )2 +(α3 −α1 )2
in terms of the coefficients of the equation a0 x3 + 3a1 x2 + 3a2 x + a3 = 0.

Solution. The given equation has the binomial coefficients with the numerical coefficients
i.e., 1, 3, 3, 1.

3a1
α1 + α2 + α3 = −
a0
3a2
α1 α2 + α2 α3 + α3 α1 =
a0
a3
α1 α3 α2 = .
a0
5.8. THEOREMS RELATING TO SYMMETRIC FUNCTIONS 99

Consider

(α1 − α2 )2 + (α2 − α3 )2 + (α3 − α1 )2 = α12 + α22 − 2α1 α2 + α22 + α32 − 2α2 α3 + α32 + α12 − 2α1 α3
= 2α12 + 2α22 + 2α32 − 2(α1 α2 + α2 α3 + α1 α3 )
= 2(α12 + α22 + α32 ) − 2(α1 α2 + α2 α3 + α1 α3 )
= 2(α1 + α2 + α3 )2 − 4(α1 α2 + α2 α3 + α1 α3 ) − 2(α1 α2 + α2 α3 + α1 α3 )
3a1 2 a2
= 2(− ) − 6(3 )
a0 a0
a2 a2
(α1 − α2 )2 + (α2 − α3 )2 + (α3 − α1 )2 = 18 21 − 18
a0 a0
2 2 2 2 2
⇒ a0 (α1 − α2 ) + (α2 − α3 ) + (α3 − α1 ) = 18(a1 − a0 a2 ).

Example 5.10. Express (2α1 − α2 − α3 )(2α2 − α1 − α3 )(2α3 − α1 − α2 ) in terms of


coefficients of the equation a0 x3 + 3a1 x2 + 3a2 x + a3 = 0.
Solution.
3a1
2α1 − α2 − α3 = 3α1 − α1 − α2 − α3 = 3α1 + ,
a0
3a1
2α2 − α1 − α3 = 3α2 + ,
a0
3a1
2α3 − α1 − α2 = 3α3 + .
a0

3a1 3a1
(2α1 − α2 − α3 )(2α2 − α1 − α3 )(2α3 − α1 − α2 ) = (3α1 + )(3α2 + )
a0 a0
3a1
+ (3α3 + )
a0
a1 a1 a2 a1
= (9α1 α2 + 9α1 + 9α2 + 9 21 )(3α3 + 3 )
a0 a0 a0 a0
2
a1 a
= 27α1 α2 α3 + 27α1 α2 + 27α1 α3 12
a0 a0
2 2 2
a a a a2 a3
+ 27α1 12 + 27α2 α3 12 + 27α2 12 + 27α3 12 + 27 13
a0 a0 a0 a0 a0
2
a3 a
= 27 + 27 21 (α1 α2 + α2 α3 + α3 α1 )
a0 a0
2
a a3
+ 27 21 (α1 + α2 + α3 ) + 27 13
a0 a0
2
a3 a1 a2 a21 a1 a31
= 27 + 27 2 (3 ) + 27 2 (−3 ) + +27 3
a0 a0 a0 a0 a0 a0
2 3
a3 a a2 a
= 27 + 81 21 ( ) − 27(2 13 )
a0 a0 a0 a0
⇒ a0 (2α1 − α2 − α3 )(2α2 − α1 − α3 )(2α3 − α1 − α2 ) = 27(a3 a20 + 3a21 a0 − 2a31 ).
3
100 LESSON - 5. SYMMETRIC FUNCTIONS

5.9 Computation of Symmetric Functions


P
Sometimes -functions involves large number of roots with small exponents,
P in such cases
simpler symmetric functions
P 2are2 multipliecd together to obtain the -functions.
For example,
P to P x1 x2 x3 x4 , we have
find
E2 E4 = x1 x2 · x1 x2 x3 x4 .

5.10 Summary
5.11 Self Assessment Excercise
1. If α1 , α2 , α3 are the roots of the equation x3 − px2 + qx − r = 0, find the values of
P  α1 α2 
(a) α2
+ α1 .
P 2
(b) α1 .
P 3
(c) α1 .

2. If α1 , α2 , α3 are the roots of the equation x3 − px2 + qx − r = 0, find the values of


P α21 +α22
α1 +α2
.

3. If α1 , α2 , α3 are the roots of the equation x3 + px + q = 0, find the values of

(α1 + α2 )2
P
(a)
P 1
(b) α1 +α2
P 2 2
(c) α1 α2
P  α1 α2 
(d) α2
+ α1
Lesson - 6

Transformation by Symmetric Functions

Structure
6.1 Learning Objectives 101
6.2 Introduction 101
6.3 Transformation by Symmetric Functions 102
6.4 Transformation in General 104
6.5 Equation of Differences in General 106
6.6 Summary 108
6.7 Self Assessment Exercise 108

6.1 Learning Objectives


After studying this chapter, student should be able to

• define symmetric functions and can check whether the given function is symmetric
or not.

• implement the fundamental theorem on symmetric functions.

• compute symmetric functions.

• better understanding of transformation of equations with the help of symmetric func-


tions.

6.2 Introduction
In algebra, theory of equations is the study of algebraic equations given by a polynomial.
Around 18th century, there were two questions of major interest first was the roots of
the equation and second was the relationship between the roots and the coefficients of the
equation. Theory of equations are also studied through the special type of functions called

101
102 LESSON - 6. TRANSFORMATION BY SYMMETRIC FUNCTIONS

symmetric functions. The theory of Symmetric functions play an important role in the
representation theory of Groups and Combinatorics. Many mathematicians uses symmetry
function theory to study the permutations and cycle structure. It has many applications in
mathematics and mathematical physics like lie algebra, random matrix theory and symme-
tries.

6.3 Transformation by Symmetric Functions


Sometimes without knowing the roots of an equation in terms of its coefficients, we can
transform one symmetric equation into another in which roots of the new equation has
some relation with the roots of the previous equation.
Suppose it is required to transform an equation into another whose roots are rational
functions of the roots of the given equation. Let the given function ϕ(α1 , α2 , α3 , · · · )
contain all the roots. The transformed equation consist of all the possible combinations
ϕ(α1 , α2 , α3 ), ϕ(α1 , α2 , α4 ) etc given by

(y − ϕ(α1 , α2 , α3 )(y − ϕ(α1 , α2 , α4 ) = 0.

Example 6.1. If α1 , α2 , α3 are the roots of x3 + ax2 + bx + c = 0, find the equation whose
roots are α12 , α22 , α32 .

Solution. Our aim is to find the equation whose roots are square of the roots of

x3 + ax2 + bx + c = 0.

Suppose the transformed equation be y 3 + Ax2 + Bx + C = 0. Then we have

−A = α12 + α22 + α32 ,


X
B= α12 α22 ,
−C = α12 α22 α32 .
P 2 P 2 2
Now, we have to form the symmetric functions α1 , α1 α2 of the given equation. We
obtain
X
α12 = a2 − 2b,
X
α12 α22 = b2 − 2ac,
α12 α22 α32 = c2 .

Therefore, the transformed equation is


y 3 − (a2 − 2b)y 2 + (b2 − 2ac)y − c2 = 0..

Example 6.2. If α1 , α2 , α3 are the roots of x3 + ax2 + bx + c = 0, find the equation whose
roots are α13 , α23 , α33 .
6.3. TRANSFORMATION BY SYMMETRIC FUNCTIONS 103

Solution. Our aim is to find the equation whose roots are cube of the roots of

x3 + ax2 + bx + c = 0.

Suppose the transformed equation be y 3 + Ax2 + Bx + C = 0. Then we have

−A = α13 + α23 + α33 ,


X
B= α13 α23 ,
−C = α13 α23 α33 .
P 3 P 3 3
Now, we have to form the symmetric functions α1 , α1 α2 of the given equation. We
obtain
X
α13 = a3 − 3ab + 3c,
X
α13 α23 = b3 − 3abc + 3c2 ,
α13 α23 α33 = −c3 .

Therefore, the transformed equation is


y 3 − (a3 − 3ab + 3c)y 2 + (b3 − 3abc + 3c2 )y + c3 = 0..
Example 6.3. If α1 , α2 , α3 , α4 are the roots of x4 + ax3 + bx2 + cx + d = 0, find the
equation whose roots are α12 , α22 , α32 , α42 .
Solution. Our aim is to find the equation whose roots are square of the roots of

x4 + ax3 + bx2 + cx + d = 0.

Suppose the transformed equation be y 3 + Ax3 + Bx2 + Cx + D = 0. Then we have

−A = α12 + α22 + α32 ,


X
B= α12 α22 ,
X
−C = α12 α22 α32 α42 ,
D = α12 α22 α32 α42 .
P 2 P 2 2 P 2 2 2 2 2 2 2
Now, we have to form the symmetric functions α1 , α1 α2 , α1 α2 α3 , α1 α2 α3 α4 of
the given equation. We obtain
X
α12 = a2 − 2b,
X
α12 α22 = b2 − 2ac + 2d,
X
α12 α22 α32 = c2 − 2bd,
α12 α22 α32 α42 = d2 .

Therefore, the transformed equation is


y 4 − (a2 − 2b)y 3 + (b2 − 2ac + 2d)y 2 − (c2 − 2bd)y + d2 = 0.
104 LESSON - 6. TRANSFORMATION BY SYMMETRIC FUNCTIONS

Example 6.4. Form an equation whose roots are the squares of the roots of the equation
x3 + x2 + 7x + 4 = 0.

Solution. Let y = x2 . The given equation may be written as

y · x + y + 7x + 4 = 0
⇒ y · x + 7x = −y − 4
⇒ x(y + 7) = −1(y + 4). (6.1)

Squaring both the sides of (3.1), we get

x2 (y + 7)2 = (y + 4)2
⇒ y(y + 7)2 = (y + 4)2
⇒ y(y 2 + 14y + 49) = y 2 + 8y + 16
⇒ y 3 + 14y 2 + 49y = y 2 + 8y + 16
⇒ y 3 + 13y 2 + 41y − 16 = 0.

Hence, y 3 + 13y 2 + 41y − 16 = 0 is the required equation.

In-text Exercise 6.1. 1. If α1 , α2 , α3 are the roots of the equation x3 − ax2 + bx + c = 0


then form an equation whose roots are α12 , α22 , α32 .

2. If α1 , α2 , α3 are the roots of the equation x3 −ax2 +bx−c = 0 then form an equation
whose roots are α13 , α23 , α33 .

3. If α1 , α2 , α3 are the roots of the equation x3 − px2 + qx2 − rx + s = 0 then form an


equation whose roots are α12 , α22 , α32 , α42 .

4. If α1 , α2 , α3 are the roots of the equation x3 − px2 + qx2 + rx + s = 0 then form an


equation whose roots are α13 , α23 , α33 , α43 .

5. Find the equation whose roots are the cubes of the roots of the equation x3 + x2 −
4x + 6 = 0.

6. Find the equation whose roots are the squares of the roots of the equation x4 − 5x −
7 = 0.

6.4 Transformation in General


In general case, aim to find a new equation where the roots of the new equation are related
by a given relation g(x, y) = 0 to the roots of the given equation f (x) = 0. We will
obtain the transformed equation by substituting the value of x in terms of y with the help
of g(x, y) = 0.

Example 6.5. If α1 , α2 , α3 are the roots of x3 − bx2 + cx − d = 0, find the equation whose
roots are α2 α3 + α11 , α3 α1 + α12 , α1 α2 + α13 .
6.4. TRANSFORMATION IN GENERAL 105

Solution. Let y = α2 α3 + α11 = α1 αα2 α1 3 +1 = d+1


α1
.
Thus we have y = x . This gives that x = d+1
d+1
y
. Substituting this value of x in the
given equation, we get
 3  2  
d+1 d+1 d+1
−b +c −d=0
y y y
⇒ (d + 1)3 − b(d + 1)y + c(d + 1)y 2 − dy 3 = 0
⇒ dy 3 − c(d + 1)y 2 + b(d + 1)y − (d + 1)3 = 0.

Hence, dx3 − c(d + 1)x2 + b(d + 1)x − (d + 1)3 = 0 is the required equation.
Example 6.6. If α1 , α2 , α3 are the roots of x3 + ax2 + bx + c = 0, find the equation whose
roots are α1 − α21α3 , α2 − α31α1 , α3 − α11α2 .

Solution. Let y = α1 − α21α3 = α1 − α1 α1 α12 α3 = α1 1 + 1c .



cy
Thus we have y = x(1 + 1c ). This gives that x = 1+c . Substituting this value of x in the
given equation, we get
 3  2  
cy cy cy
+a +b +c=0
1+c 1+c 1+c
⇒ (cy)3 + ac2 (1 + c)y 2 + bc(1 + c)y 2 − cy 3 = 0
⇒ c2 y 3 + ac(1 + c)y 2 + b(1 + c)2 y + (1 + c)3 = 0.

Hence, c2 x3 + ac(1 + c)x2 + b(1 + c)2 x + (1 + c)3 = 0 is the required equation.


Example 6.7. Find the value of α12 + α22 + α32 for the given cubic equation

x3 − ax2 + bx − c = 0.

Solution.

α1 + α2 + α3 = −a,
α1 α2 + α2 α3 + α3 α1 = b
⇒ (α1 + α2 + α3 )2 = α12 + α22 + α32 + 2α1 α2 + 2α2 α3 + 2α3 α1
= α12 + α22 + α32 + 2(α1 α2 + α2 α3 + α3 α1 )
⇒ a2 = α12 + α22 + α32 + 2b
⇒ α12 + α22 + α32 = a2 − 2b.

Example 6.8. If α1 , α2 , α3 are the roots of the equation x3 + ax2 + bx + c = 0, then find
the equation whose roots are α2 +αα31 −α1 , α3 +αα12 −α2 , α1 +αα32 −α3 .
Solution. α1 + α2 + α3 = −b, α1 α2 α3 = −c. Take
α1 α1 α1
y= = =
α2 + α3 − α1 −a − α1 − α1 −a − 2α1
ay
⇒x=− .
1 + 2y
106 LESSON - 6. TRANSFORMATION BY SYMMETRIC FUNCTIONS

Putting the value of x in the given equation, we get


ay 3 ay 2 ay
(− ) + a(− ) + b(− )+c=0
1 + 2y 1 + 2y 1 + 2y
−a3 y 3 + a32y 2 (1 + 2y) − aby(1 + 2y)2 + c(1 + 2y)3 = 0.

In-text Exercise 6.2. 1. If α1 , α2 , α3 are the roots of the equation x3 +ex2 +f x+g = 0,
then find the equation whose roots are α12 + 2α2 α3 , α22 + 2α3 α1 , α32 + 2α1 α2 .

2. If α1 , α2 , α3 are the roots of the equation x3 −qx2 +rx−s = 0, then find the equation
whose roots are α2 α3 − α12 , α3 α1 − α22 , α1 α2 − α32 .

3. If α1 , α2 , α3 are the roots of the equation x3 + ax2 + b = 0, form an equation whose


roots are α2 + α3 , α3 + α1 , α1 + α2 .

4. If α1 , α2 , α3 are the roots of the equation x3 + kx2 + m = 0, form an equation whose


roots are αα2 α1 3 , αα3 α2 1 , αα1 α3 2 .

6.5 Equation of Differences in General


The general problem of the formulation of the equation whose roots are the differences or
the square of the differences of the roots of the given equation.

Example 6.9. If α1 , α2 , α3 are the roots of the equation x3 − 6x2 + 11x − 6 = 0 then find
the equation whose roots are α12 + α22 , α22 + α32 , α32 + α12 .

Solution. We have α1 + α2 + α3 = 6, α1 α2 α3 = 6.
Let

y = α22 + α32 (6.2)


⇒ y = (α2 + α3 )2 − 2α2 α3 (6.3)
⇒ y = (6 − α1 )2 − 12/α1
⇒ α1 y = α1 (6 − α1 )2 − 12
⇒ α13 − 12α12 + (36 − y)α1 − 12 = 0 (6.4)

Since α1 satisfies the given equation, so

α13 − 6α12 + 11α1 − 6 = 0. (6.5)

Subtracting (3.2) from (3.1), we get

6α12 + (y − 25)α1 + 36 = 0 (6.6)

Example 6.10. If α1 , α2 , α3 are the roots of the equation x3 + bx + c = 0 then find the
equation whose roots are αα12 + αα12 , αα32 + αα23 , αα13 + αα13 .
6.5. EQUATION OF DIFFERENCES IN GENERAL 107

α21 +α22
Solution. Let y = α1 α2
. Then

α1 + α2 + α3 = 0
⇒ −α3 ) = α1 + α2
⇒ (α3 )2 = (α1 + α2 )2
⇒ α32 = α12 + α22 + 2α1 α2
⇒ α32 − 2α1 α2 = α12 + α22 .
α2 −2α α α2 α3 α33
This gives that y = 3 α1 α21 2 = α1 α3 2 − 2 = α1 α32 α3 − 2 = − c
− 2. This implies that
c(y + 2) = −α33 . We take x3 = −c(y + 2).
Putting the value of x3 in the given equation, we get
− c(y + 2) + bx + c = 0
⇒bx − cy − c = 0
⇒bx = c(y + 1)
⇒b3 x3 = c3 (y + 1)3
⇒ − b3 c(y + 2) = c3 (y + 1)3
⇒ − b3 (y + 2) = c2 (y 3 + 1 + 3x2 + 3x)
⇒c2 (y 3 + 3y 2 + 3y + 1) + b3 (y + 2) = 0
⇒c2 y 3 + 3c2 y 2 + 3cy + c2 + b3 y + 2b3 = 0
⇒c2 y 3 + 3c2 y + (3c + b3 )y + c2 + 2b3 = 0.
Example 6.11. If α1 , α2 , α3 are the roots of the equation x3 + bx + c = 0 then find the
equation whose roots are lα1 + mα3 α2 + lα2 + mα3 α1 , lα3 + mα1 α2 .
α21 +α22
Solution. Let α1 + α2 + α3 = 0, α1 α2 + α2 α3 + α3 α1 = b, α1 α2 α3 = −c. y = α1 α2
.
Let
y = lα1 + mα2 α3
⇒ α1 y = lα12 + α1 α2 α3
⇒ α12 − α1 y − mc = 0 (6.7)
Since α1 is the root of the given equation, therefore
α13 + bα1 + c = 0. (6.8)
Multiplying (4) by α1 , (5) by l and subtract, we get
yα12 + (mc + lb)α1 + lc = 0. (6.9)
Solving 4,6 for α1 , α12 , we have
α12 α1 1
= 2
=
−lcy + mc(mc + lb) −mcy − l c l(mc + lq) + y 2
mc(mc + lb) − lcy −(mcy + l2 c)
⇒ α12 = , α 1 = .
l(mc + lq) + y 2 l(mc + lq) + y 2
108 LESSON - 6. TRANSFORMATION BY SYMMETRIC FUNCTIONS

Eliminating α1 , we get

c[m(mc + lb) − ly] c2 [(my + l2 )]2


=
l(mc + lq) + y 2 [l(mc + lq) + y 2 ]2

Hence, [l(mc + lq) + y 2 ][m(mc + lb) − ly] = c[(my + l2 )]2 .

6.6 Summary
6.7 Self Assessment Exercise
1. If α1 , α2 , α3 are the roots of the equation x3 − 3x2 + 5x − 11 = 0 then find the
equation whose roots are α1 + α2 , α2 + α3 , α3 + α1 .

2. If α1 , α2 , α3 are the roots of the equation x3 − 7x2 + 12x − 6 = 0, form an equation


whose roots are α12 + α12 , α12 + α12 , α12 + α12 .
1 2 2 3 3 1

3. If α1 , α2 , α3 are the roots of the equation x3 + 3x2 + 2x − 4 = 0 then find the equation
whose roots are α12 + α22 − α32 , α22 + α32 − α12 , α32 + α12 − α22 .

Suggested Reading
1. Burnside, W.S., Panton, A.W.,The Theory of Equations (11th ed.). Vol. 1. Dover
Publications, Inc., 1979.

2. Dickson, Leonard Eugene, First Course in the Theory of Equations. John Wiley and
Sons, Inc., 2009

You might also like